Haem2 Flashcards

1
Q

Which one of the following is the most important component in the pathogenesis of β thalassaemia?

A. Iron overload.

B. α/β globin chain imbalance.

C. Ineffective erythropoiesis.

D. Dysfunctional β globin.

E. Haemolytic anaemia.

A

**B. α/β globin chain imbalance.

Excess alpha chains are unstable and precipitate - RBC membrane damage (target cell)

Results in:

  • Intramedullary and peripheral haemolysis
  • Erythroid hyperplasia,
  • Ineffective erythropoeisis due to intramedullary destruction of erythroid cells.**
How well did you know this?
1
Not at all
2
3
4
5
Perfectly
2
Q
  1. What are the three different kinds of haemoglobin?
  2. Which forms the majority of adult Hb?
  3. On what genes are haemoglobin alpha and beta located?
A

HBA (A2B2)- a tetramer of two alpha and two beta chains

HBA2 (a2δ2) tetramer forms - 1-2% adult Hb

HBF - a2γ2 tetramer forms - fetal Hb but <1% of adult Hb

  1. 98% adult Hb primarily HBa

2 alpha-globin genes - Cr16, w/ no substitutes.

B-globin gene - Cr11, adjacent to B-like glob in genes delta δ and gamma γ

How well did you know this?
1
Not at all
2
3
4
5
Perfectly
3
Q

In the context of thalassaemia, define the following terms:

  1. Trait
  2. Intermedia
  3. Major
A
  1. Trait - lab features, no clinical impact
  2. Intermedia - occastional transfusion requirement
  3. Major - life threatening or transfusion dependent - most die of iron overload.
How well did you know this?
1
Not at all
2
3
4
5
Perfectly
4
Q

Describe the pathogenesis of alpha-thalassaemia

A

Alpha thalassaemia - Cr16 x 2

  • due to gene deletions —> reduced a-globin chain synthesis
  • each gene produces 1/4 of total alpha-globin quantity
  • all adult Hb are alpha containing, so there will never be any change in percentage distribution of it on Hb EPG.
  • Severe forms: excess B chains will form B2B2 tetramer (H bodies), or HbH which are highly unstable.
How well did you know this?
1
Not at all
2
3
4
5
Perfectly
5
Q

Describe the clinical features of alpha thalassaemia

A

Southeast Asians and Chinese

  • aa/a- : silent carrier
  • aa/— or a-/a- : Clinically normal, mild microcytic anaemia
  • a-/— : HbH disease - chronic haemolytic anaemia of variable severity (minor or intermedia) with pallor and splenomegaly. Intermittent transfusions only (infection / stress or by RBC production shutdown (aplastic crisis) by certain viruses (i.e. parvovirus).)
  • —/— : Hydrops fetalis (no normal Hb is produced only Bart’s y4) - fetus is stillborn
How well did you know this?
1
Not at all
2
3
4
5
Perfectly
6
Q

Describe the lab findings for alpha-thalassaemia trait and HbH disease.

A

Trait - MCV (60-75)

  • normal or increased RBC count
  • smear: microcytes, hypochromia, occ target cells, acanthocytes.
  • retic and iron counts normal
  • Hb EPG: no change in percent HbA2/F/or HbH
  • dx of exclusion

HbH disease

  • marked anaemia HCT 22-32%
  • very low MCV 60-70
  • smear: hypochromia, microcytosis, target cells, poikilocytosis (annoying term that means ‘varied’!)
  • elevated retics, elevated/normal RBC count
  • Hb EPG: fast migrating haemoglobin (HbH) accounting for 10-40% of the Hb.
  • Can stain smear with supra vital dye to show HbH
How well did you know this?
1
Not at all
2
3
4
5
Perfectly
7
Q

Describe the pathogenesis of beta-thalassaemia

A

Beta thalassamia - Cr 11 point mutation

  • little to no B-Hb –> more HBA2 & HbF on EPG
  • excess alpha chains precipitate –> RBC membrane damage
  • intramedullary and peripheral haemolysis, erythroid hyperplasia

Absent b-chains: B0

Reduced b-chains: B+ (variable severity)

Severe disease

  • marrow expansion –> bone deformities, osteopaenia, path fractures
How well did you know this?
1
Not at all
2
3
4
5
Perfectly
8
Q

Describe the clinical manifestations of all forms of beta-thalassaemia

A

Homozygotes (B0/B0 or B+/B+)

  • thal major
  • normal @ birth, severe tfusion dependence at 6/12 (HbF–>HbA)
  • Stunted growth
  • Bony deformities (frontal bossing, chipmunk facies, path fractures)
  • Hepatosplenomegaly
  • Cirrhosis, jaundice (due to gallstones), thrombophilia
  • Iron overload (due to transfusion) –> heart failure & arrythmias, cirrhosis, endocrinopathies, pseudoxanthoma elasticum - calcification of epithelial cells of skin, retina, CVS) due to inability to excrete iron. Poor survival.

Homozygosity for milder B+/B+: chronic haemolytic anaemia, transfusions only during aplastic crises. May still develop iron overload, hepatosplenomegaly and bony deformities but survive into adulthood.

Heterozygosity for B/B0 or B/B+ : thalassaemia minor, clinically insignificant microcytic anaemia

How well did you know this?
1
Not at all
2
3
4
5
Perfectly
9
Q

What are the lab findings for beta-thal major and minor?

A

B-thal minor:

  • Modest anaemia - HCT 28-40%, MCV 55-75
  • RBC count normal or increased
  • Retic count - normal or slightly elevated
  • Smear: Hypochromia, microcytosis, target cells, basophilic stippling
  • Hb EPG: Increased HBA2 to 4-8%, occ increased HbF - 1-5%

B-thal major:

  • severe anaemia - HCT to <!0% w/o transfusion
  • smear: severe poikilocytosis, hypochromia, microcytosis, target cells, basophilic stiplling, nucleated RBCs.
  • HbEPG - little to no HbA, variable amt HBa2, major Hb present is HbF
How well did you know this?
1
Not at all
2
3
4
5
Perfectly
10
Q

How do you differentiate thalassaemia from iron deficiency anaemia?

How is the diagnosis of b-thal and a-thal made?

A

Compared to Fe+ deficiency anaemia

  • lower MCV
  • normal or elevated RBC count
  • more abnormal smear at modest lvld anaemia
  • normal or elevated transferrin sat/ferritin (or both)

Dx of b-thal made with above plus HbEPG showing elevated HbA2 and F (assuming iron replete)

Dx of a-thal is one of exclusion since HbEPG levels of a-globin remains the same.

Other microcytic anaemia with normal or elevated RBC count is Fe+ deficiency + PCV

How well did you know this?
1
Not at all
2
3
4
5
Perfectly
11
Q

Describe the treatment for all forms of thalassaemia

A

Treatment

Mild a or b - no treatment but need identification so they don’t keep getting investigated for Fe deficiency anaemia!

HbH - Folic acid, avoid medicinal iron and oxidative drugs (sulphonamides)

Severe thalassaemia - regular transfusion, folic acid and iron chelation.

Allogeneic SCT - treatment of choice for B-thal major, only cure, and in kids w/o Fe+ overload or chronic organ toxicity - long term survival of >80%.

How well did you know this?
1
Not at all
2
3
4
5
Perfectly
12
Q

Which one of the following occurs most frequently in the α thalassaemia syndromes?

A. Mental retardation.

B. Reduced mean corpuscular volume.

C. Gene deletions.

D. Haemoglobin H inclusion bodies.

E. Reduced haemoglobin.

A

C. Gene deletions.

How well did you know this?
1
Not at all
2
3
4
5
Perfectly
13
Q

AMP 1999 Question 30

In which one of the following conditions has initial therapy with imatinib been shown to cause the best improvement in survival?

A. Multiple myeloma.

B. Low grade non-Hodgkin’s lymphoma.

C. Chronic myeloid leukaemia.

D. Chronic lymphocytic leukaemia.

E. T-cell acute lymphocytic leukaemia.

A

C. CML

TKIs (Imatinib, Dasatinib, Nilotinib, Ponatinib) • specific competitive binders in the ATP-pocket of BCR-ABL tyrosine kinase. Associated with >80% progression free at 8 years.

How well did you know this?
1
Not at all
2
3
4
5
Perfectly
14
Q

What is the pathogenesis of CML?

A

Chromosomal abnormality

Reciprocal translocation of Cr9 to 22

Cr9q Abl protoncogene fused to BCR on Cr22q

then

Molecular abnormality

Fusion gene deregulates abl tyrosine kinase activity —> autophosphyrlation, altered adhesion, inhibition of apoptosis

then

Myeloid proliferation

How well did you know this?
1
Not at all
2
3
4
5
Perfectly
15
Q

What are the clinical findings of CML?

A
  • Middle aged, more men
  • May be asymptomatic - discovered incidentally on an FBC with elevated WCC

• Hyper metabolic sx; night sweats, low-grade fever caused by overproduction of white cells

  • May have splenomegaly and or sternal tenderness from marrow overexpansion
  • Rarely; leukostasis: blurred vision, resp distress, priapism (usually w/ very high white cell counts
  • Acceleration: Fever in absence of infection, bone pain and splenomegaly.
How well did you know this?
1
Not at all
2
3
4
5
Perfectly
16
Q

What are the lab findings in CML?

A
  • Early CML: normal BM function, normal WBCs, normal neutrophils
  • Untreated: accelerated then acute blast phase - indistinguishable from acute leukaemia
  • Median WBC count >150
  • Blood film: Left-shifted (immature cells present) with cells present in proportion to their degree of maturation (i.e. blasts <5%), normal RBC morphology, normal or elevated platelets (abnormal megakaryocytes)
  • Basophilia and eosinophilia of granulocytes is pathognomic
  • Peripheral blood PCR (required for diagnosis): Increased levels of BCR-ABL gene.
  • BM: Hypercellular w/ left shift myelopoesis
How well did you know this?
1
Not at all
2
3
4
5
Perfectly
17
Q

What is the diagnostic criteria for CML?

A
  • Peripheral blood: bcr/able gene present on PCR - hallmark of the disease – this is diagnostic criterai
  • BMB is NOT necessary for dx but useful for prognosis and detecting additional chromosomal abnormalities
How well did you know this?
1
Not at all
2
3
4
5
Perfectly
18
Q

How is the blast phase of CML defined?

What is it associated with?

A

One or more of:

o Blasts composing >20% of nucleated bone marrow cells.

o Extramedullary blast proliferation

o Large foci or clusters of blasts in the bone marrow biopsy

Evolution to a blast cell crisis in CML associated w/:

o Non-random secondary chromosomal changes

o Mutations or deletions of tumour suppressor genes

How well did you know this?
1
Not at all
2
3
4
5
Perfectly
19
Q

How do you differentiate CML from infection and other blood disorders?

A
  • Infection - no splenomegaly, WBC <50, no bcr/abl gene
  • Other blood disorders - normal RBCs, no nucleated RBCs, definitive finding of bcr/abl gene
How well did you know this?
1
Not at all
2
3
4
5
Perfectly
20
Q

Define the types and goals of therapy in CML

A

Treatment

  • Untreated progresses from chronic phase (3-5 years) to blast crisis
  • Chronic-phase CML: TKI aiming for complete remission w/ normalisation of blood count and splenomegaly within 3 months initiation. Aim for MAJOR molecular response in 1 year.
  • Blast/accelerated phase: ASCT + myelosuppressive therapy
  • Hyperleukocytosis (priapism, respiratory distress, visual blurring, altered mental state): emergency leukapheresis + myelosuppressive therapy.
How well did you know this?
1
Not at all
2
3
4
5
Perfectly
21
Q

What do the terms major molecular response, and complete molecular response refer to in the context of CML?

A

Major molecular response

  • 3-log reduction of bcr/abl transcript on PCR within a year (corresponds to bcr/abl RATIO (compared to alb) of <0.01).
  • Goal is to achieve MMR within a year
  • excellent prognosis - up to 100% progression-free at 8 years.

Complete MR = >4.5 log reduction ie undetectable

How well did you know this?
1
Not at all
2
3
4
5
Perfectly
22
Q

What is the mechanism of action of tyrosine kinase inhibitors?

What are the side effects?

A

Tyrosine Kinase Inhibitors

  • specific competitive binders in the ATP-pocket of BCR-ABL tyrosine kinase
  • Imatinib: first line, MMR @ 30% at 1 year in chronic phase. SEs: NV, chronic diarrhea, intolerable fatigue
  • Dasatinib: pleural effusions esp. in the elderly, pneumonitis, pulmonary hypertension, bleeding, possibly increased infection risk
  • Nilotinib: cardiac SEs: esp IHD, CVA, PAD (inc limb ischaemia, claudication), also increases cholesterol and LDL
  • 3rd gen agents may be more effective - can salvage 90% of patients who do not respond to imatinib. Are now approved as first line agents.
How well did you know this?
1
Not at all
2
3
4
5
Perfectly
23
Q

What is the mechanism of tyrosine kinase inhibitor resistance?

How is it managed in CML?

A

• Due to:

  • drug influx/efflux due to low OCT-1 expression
  • amplification and over expression of BCR-ABL
  • Mutations of BCR-ABL kinase domain (found in 50-90% w/ secondary resistance)
  • can increase dose with variable response (depends on mutation
  • or use alternative TKIs
How well did you know this?
1
Not at all
2
3
4
5
Perfectly
24
Q

How is disease progression in CML monitored?

A

Peripheral blood PCR, if it increases on therapy check compliance, check for a new abl mutation, may be able to get away with increased dose of Imatinib, otherwise switch to a new inhibitor.

How well did you know this?
1
Not at all
2
3
4
5
Perfectly
25
Q

What is the course and prognosis of CML?

A

Course & Prognosis

  • 80% w/o progression at 9 years w/ TKIs
  • essentially 100% survival at 9 years if good molecular response
  • Complete (undetectable bcr/abl) response lasting > 2years may be able to cease therapy all together.
  • Untreated progresses from chronic phase (3-5 years) to blast crisis
How well did you know this?
1
Not at all
2
3
4
5
Perfectly
26
Q

What is the pathogenesis of CLL?

A

Pathogenesis

  • B Cell lymphocytosis w/ coexpression of CD19, CD5 (pathognomic: aberrant T-cell marker) on lymphocytes.
  • indolent, long lived small (unactivated) lymphocytes
  • immunoincompetent, poorly antigenic responsive
  • immunosuppression,
  • BM failure
  • organ infiltration with lymphocytes
  • inadequate antibody production
  • damage due to direct tissue introduction
How well did you know this?
1
Not at all
2
3
4
5
Perfectly
27
Q

What is the epidemiology of CLL?

A

Epidemiology

  • 25% of all leukemias (2.7/100,000) , increasing
  • Unknown aetiology
  • Increased other malig (SCC)
  • Only leukemia NOT linked to radiation
  • M:F=2:1, less in Asian’s
  • Disease of old age - median at presentation is age 70
How well did you know this?
1
Not at all
2
3
4
5
Perfectly
28
Q

What are the clinical findings in CLL?

A

Clinical Findings

  • incidental: lymphocytosis - usually indolent
  • symptomatic: fatigue or lymphadenopathy
  • 80% will have lymphadenopathy, 50% splenomegaly
  • subtype prolymphocytic leukaemia (larger and more immature cells) is more aggressive but rare
  • maybe complicated by AIHA or autoimmune thrombocytopaenia
  • 5% of systemic cases may have an isolated lymph node transform into an aggressive large cell lymphoma (Richter syndrome)
How well did you know this?
1
Not at all
2
3
4
5
Perfectly
29
Q

What is the Rai staging system used for? Can you describe it?

A

Used to stage CLL

Staging - Rai

Stage 0 - lymphocytosis (low-risk)

Stage II - organomegaly (low risk)

Stage III - anaemia (high risk)

Stage IV - thrombocytopaenia (high risk)

How well did you know this?
1
Not at all
2
3
4
5
Perfectly
30
Q

What is the diagnostic criteria for CLL?

A

Diagnostic criteria

Peripheral absolute B lymphocyte count > 5 x 109/L w/ mainly morphologically mature lymphocytes

Flow cytometry: clonality of circulating B-lymphocytes w/ low levels of SmIg and either kappa or lambda (not both) and expression of B-cell antigens (CD19, 20, 23), and expression of T-Cell associated antigen CD5.

How well did you know this?
1
Not at all
2
3
4
5
Perfectly
31
Q

Name the investigations you would do when working up CLL

A

Ix

  • FBE,differential; lymphocytosis,cytopenias
  • Flowcytometry: CD19/5,Ig(weak),CD23
  • Coombes test
  • LDH, B2 microglobulin
  • Uric acid, calcium
  • BMBx – cytopenias– ‘assessment’ pre-treatment – cytogenetics
  • +/-CTscan
How well did you know this?
1
Not at all
2
3
4
5
Perfectly
32
Q

Describe the lab findings in CLL for the

FBC

Smear

BMBx

Cytogenetics.

A

Lab Findings

  • hallmark: isolated lymphocytosis
  • WBC >20k, >75% small and mature lymphocytes, normal HCT and platelets.
  • may have cytopaenias inc neutropaenia.
  • smear: normal-looking small lymphocytes predominate with reduced numbers of larger activated lymphocytes, SMUDGE CELLS
  • BMBx: Infiltrated w/ small lymphocytes

Immunophenotype (Flow): Coexpression of CD19 with T-lymphocyte marker CD5 - ONLY found in CLL and mantle-cell lymphoma (Not mantle cell lymphoma if: expression of CD23, low expression of surface Ig and CD20, absence of cyclin D1)

Cytogenetics (BMBx)

  • Associated mutation of Ig (Vh) gene = more indolent disease
  • Genomic changes assessed by flow cytometry or FISH

o Chromosome 17p (p53) deletion or 11q (ATM) = POOR prognosis

o Isolated 13q deletion = favourable prognosis (in Myeloma, it’s BAD!)

• 50% have hypogammaglobulinaemia, more common with advanced disease, may have a small amt of IgM paraprotein.

How well did you know this?
1
Not at all
2
3
4
5
Perfectly
33
Q

Describe the chromosomal abnormalities that are associated with good and poor prognoses in CLL

A

o Chromosome 17p (p53) deletion or 11q (ATM) = POOR prognosis

o Isolated 13q deletion = GOOD prognosis (in Myeloma, it’s BAD!)

How well did you know this?
1
Not at all
2
3
4
5
Perfectly
34
Q

DDx of CLL?

A

DDx

  • Exclude viral infections that cause lymphocytosis (i.e. Pertussis)
  • Fever in CLL may be concomitant bacterial infection
  • Other B-Cellopathies are distinguished on morphology and immunophenotype.
  • Monoclonal B-cell lymphocytosis (<5x109) is a precursor to B-CLL
How well did you know this?
1
Not at all
2
3
4
5
Perfectly
35
Q

Describe the treatment approaches and what you would use for each of the following groups in CLL:

Comorbid elderly

Robust older patient

Younger patient

A

Comorbid elderly - symptom control, QOL —> observe or ‘gently oral CTx with chlorambucil or cyclophosphmade

Robust older pt - QOL, bulk reduction, deepest achievable response —> observe is asymptomatic, otherwise multi drug CTx, ritux + fludarabine for prolonged remission.

Younger pt - as above, possible allogeneic BMTc

How well did you know this?
1
Not at all
2
3
4
5
Perfectly
36
Q

What prophylaxis should you use when giving fludarabine or alemtuzumab in CLL?

A

Prophylaxis

  • Fludarabine or alemtuzumab —> PJP pneumonia, HSV, HZV and fungal prophylaxis.
  • Alemtuzumab - monitor for CMV reactivation
  • Lower strength ‘nonmyeloablative’ transplant + drugs good for older people.
How well did you know this?
1
Not at all
2
3
4
5
Perfectly
37
Q

What are the prognostic features of CLL?

A

Prognostic features

Stage 0-1: 10-15 years,

Stage 3-4: >90% at 2 years w/ fludarabine

High risk disease: Allogeneic transplant may lead to long term disease control.

Serum LDH and B2-microglobulin

Unmutated IgH (adverse) vs mutated

  • ZAP 70 (zeta chain associated protein) (bad if positive) , CD38 + (bad if +)
  • Cytogenetic translocations (not trisomies or deletions)
  • ~5% terminally  “Richter’s syndrome”; consider if: B-Sx, painful, rapidly enlarging mass rapid unexpected rise in LDH
How well did you know this?
1
Not at all
2
3
4
5
Perfectly
38
Q

What is Richters Syndrome?

A

Transformation of CLL

5-10% of CLL patients

B-Sx, painful, rapidly enlarging nodal mass and rapid unexpected rise in LDH

fast-growing diffuse large B cell lymphoma

BAD prognosis

How well did you know this?
1
Not at all
2
3
4
5
Perfectly
39
Q

What is alemtuzumab and when is it used?

A

Alemtuzumab (mAb to mature lymphocyte CD52), but immunosuppression, severe and fatal infections.

Used in CLL

How well did you know this?
1
Not at all
2
3
4
5
Perfectly
40
Q

What is Ibrutinib and when is it used?

What is a Bruton’s tyrosine kinase?

A

Used in CLL

Ibrutinib (PCI-02765) a Bruton’s TKI.

Bruton’s tyrosine kinase (BTK) BTK is a kinase that plays a crucial role in B-cell development

How well did you know this?
1
Not at all
2
3
4
5
Perfectly
41
Q

How do you treat AIHA in CLL?

A

AIHA: rituximab, prednisone or splenectomy - AVOID fludarabine as may exacerbate (but concurrent rituximab reduces this risk)

How well did you know this?
1
Not at all
2
3
4
5
Perfectly
42
Q

What is the pathogenesis of multiple myeloma?

A

Monoclonal proliferation (kappa/lambda light chain restriction) of plasma cells in the BM (any percentage) or as a tumor (plasmacytoma) resulting in:

End-organ damage:

C - Hypercalcaemia

R - Renal failure

A - Anaemia

B - Bone disease

Recurrent infections w/ encapsulated organisms (due to b-cell deficiency)

Plasmacytomas

Hyperviscosity syndrome

How well did you know this?
1
Not at all
2
3
4
5
Perfectly
43
Q

Who is most susceptible to overwhelming infection with encapsulated organisms?

A
Asplenic patients
B-cell deficient patients
Complement deficiency (esp C3 and C5+ (MAC complex)

Children 6/12 - 1 year

How well did you know this?
1
Not at all
2
3
4
5
Perfectly
44
Q

Name the encapsulated organisms.

Why are they important?

A

Streptococcus pneumoniae

Haemophilus influenzae type B (B polysaccharide)

Neisseria meningitidis

Klebsiella pneumoniae

Salmonella typhi

Pseudomonas aeruginosa

Cryptococcus neoformans

only encapsulated fungal pathogen

Other encapsulated bacteria

Anti-phagocytic - susceptible to antibody (B-Cells) and not cell mediated responses. People w/ B-cell deficiencies are highly susceptible.

How well did you know this?
1
Not at all
2
3
4
5
Perfectly
45
Q

What is the mechanism and outcome of hyperviscosity syndrome in multiple myeloma?

A

Excess paraprotein production (monoclonal Igs) in addition to excess plasma cells —> hyperviscosity —> spont.bleeding, retinopathy/vision disturb, vertigo, nausea, seizures, coma

How well did you know this?
1
Not at all
2
3
4
5
Perfectly
46
Q

What is the mechanism of renal failure in multiple myeloma?

A

Cast nephropathy: Tumour mass expansion —> FLC serum concentration exceeds renal absorptive capacity →excess light chains—> direct damage to PCT—> casts at distal tubules (containing BJP and tamm-horsfall proteins) —> FLC in urine, positive BJP

Monoclonal Ig deposition disease (usually light chains)—> glomerular disease, proteinuria

Hypercalcaemia —>nephrocalcinosis

Cryoglobulins, Fanconi syndrome (type2 RTA, phosphaturia, glycosuria, uricosuria & osteomalacia), hyperuricemia

How well did you know this?
1
Not at all
2
3
4
5
Perfectly
47
Q

What is the mechanism of recurrent infection in multiple myeloma?

A

Underproduction of normal Ig + neutropenia + CTx immunosuppression —> recurrent infections esp encapsulated organisms.

How well did you know this?
1
Not at all
2
3
4
5
Perfectly
48
Q

What are the clinical findings of multiple myeloma and who does it affect?

A

Older adults - i.e. 65 y old

Pallor, bone tenderness, soft tissue masses

Bone tenderness/pain: hips, back, ribs, path # esp fem neck, vertebrae

Neurologic signs: neuropathy, SC compression (plasmacytoma)

Primary amyloid —> enlarged tongue, peripheral/autonomic neuropathy, CHF, hepatomegaly

NO splenomegaly unless amyloidosis

NO fever unless infection

How well did you know this?
1
Not at all
2
3
4
5
Perfectly
49
Q

What are the lab findings of multiple myeloma?

What is the predominant paraprotein?

A

FBE: Rouleaux (high protein), cytopenias, macrocytosis

ESR elevated

Normal urine ‘dip-stick’ (i.e BJP): may be + if nephrotic (albumin)

Electrolytes: Renal function, calcium, ALP normal

Low B12

Protein EPG (serum or urine) - detects a clonal protein

  • Monoclonal spike in b or gamma globulin region, known as ‘M-protein’

IEPG - identifies the clonal protein

  • Paraprotein estimation: Serum/urine: IgG(70%) >IgA(20%) >IgD(5%) >LCD(5%) >NS(2.5) >>IgM
  • no paraprotein = nonsecretory myeloma = BAD

Elevated B2 microglobulin (prognostic test)

How well did you know this?
1
Not at all
2
3
4
5
Perfectly
50
Q

What are free light chains and what is the role of the serum free light chain test?

A

Free Light Chains

  • Ig are heavy chains – the light chains (kappa or lambda) produced separately
  • kappa and lambda bound to heavy chains, but free light chains produced in healthy people when there are more light chains than heavy chains
  • bound and free light chains are structurally identical
  • Free light chains reach a threshold before detectable in urine - need for serum FLC.
  • 15% will have no paraprotein due to free light chain only disease which pass freely into the urine.

Serum FLC (free light chains)

Will pick up light chains in non-secretory myeloma (too low to be detected in urine)

  • use at dx of all types of myeloma inc MGUS, plasmacytoma and AL amyloidosis
  • use in combo with EPG/IEPG - sufficient to screen for all except AL amyloid (which needs more serum tests and urine IEPG)
How well did you know this?
1
Not at all
2
3
4
5
Perfectly
51
Q

When would you do a bone marrow in multiple myeloma?

What would it show?

How do you differentiate it from reactive (benign) plasmacytosis?

A

Bone marrow (only for symptomatic - i.e. paraprotein plus end organ damage)

Morphologically abnormal plasma cells

Skewing of kappa to lambda light chain ratio

May have reactive (benign) plasmacytosis - but atypical cells, LC restriction and effacement of normal BM = myeloma

How well did you know this?
1
Not at all
2
3
4
5
Perfectly
52
Q

In multiple myeloma, when would you do a:

Skeletal survey

Nuclear bone scan

MRI/PET?

and what would they show?

A

Symptomatic myeloma: Skeletal survey (whole body x-ray)

  • lytic lesions of axial skeleton; skull, spine, prix long bones, ribs
  • OR general OP only

Wouldn’t - Nuclear bone scan - unuseful, no osteoblastic component

MRI - severe back pain, suspected vertebral compression, isolated plasmacytoma

  • really good but expensive, pick up lesions that x-ray doesn’t

PET - pre CTx

How well did you know this?
1
Not at all
2
3
4
5
Perfectly
53
Q

What is the differential diagnosis of a positive paraprotein on EPG?

A

CLL

Waldenstrom

NHL

primary amyloid

cryoglubinaemia

MGUS

How well did you know this?
1
Not at all
2
3
4
5
Perfectly
54
Q

What is the definition and risk factors for progression of MGUS to MM?

A

BM monoclonal plasma cells <10% or serum M-protein <30g/L)

No end organ damage or other B-cell disorder

1/4 progresses to malignant disease in 10 years (1%/year)

Must be distinguished from reactive (benign) polyclonal hypergammaglobulinaemia

Risk factors for progression

Size of M-band (best predictor of risk)

IgM or IgA monoclonal protein

Eleveated serum FLC

How well did you know this?
1
Not at all
2
3
4
5
Perfectly
55
Q

What is the definition of smoldering myeloma?

A

BM monoclonal plasma cells >10% or serum M-protein >30g/L

No end-organ damage

How well did you know this?
1
Not at all
2
3
4
5
Perfectly
56
Q

What is the treatment for MGUS and smoldering myeloma?

A

Observe

How well did you know this?
1
Not at all
2
3
4
5
Perfectly
57
Q

What is the role of auto-sct and thalidomide in multiple myeloma?

A

Auto-SCT

  • prolongs duration of remission and overall survival, long treatment-free intervals
  • thalido/lenalidomide prolong remission and survival as post-transplant maintenance therapy
How well did you know this?
1
Not at all
2
3
4
5
Perfectly
58
Q

What is the treatment of symptomatic myeloma in the non-transplant candidate?

A

Non-transplant candidate: Melphalan + Pred + Thalidomide > Melphalan, Pred, bortezomib

How well did you know this?
1
Not at all
2
3
4
5
Perfectly
59
Q

What is the treatment of symptomatic multiple myeloma in an eligible candidate?

A

–Auto-Transplant ‘candidate’ (<76)

» Induction therapy: chemotherapy +/- thalidomide, bortezomib, lenalidomide

» Melphalan (alkylator) stem cell transplant [tandem v single]

» Maintenance: steroids, thalidomide

How well did you know this?
1
Not at all
2
3
4
5
Perfectly
60
Q

What is bortezomib, when is it used and it’s side effects?

A

Bortezomib (proteosome inhibitor)

Relapsed MM /poor prognosis –> rapid response.

SE: neuropathy (peripheral/autonomic), less if given SC (instead of IV)

How well did you know this?
1
Not at all
2
3
4
5
Perfectly
61
Q

What is thalidomide/lenalidomide, when is it used and it’s side effects?

A

Inhibits angiogenesis

Used for multiple myeloma and leprosy

Thalidomide/Lenalidomide SEs

  • Somnolence
  • Thalidomide neuropathy – painful neuropathy, reduced sens nerve AP (AP)
  • Constipation, tremor, skin rash and oedema
  • Thrombosis
  • Myelosuppression
How well did you know this?
1
Not at all
2
3
4
5
Perfectly
62
Q

Define the international staging system - what is it used for?

A

International Staging System

B2 microglobulin (both stages) and albumin (stage 1)

Stage 1 - low B2 (<3.5mg/L), normal albumin - survival >5 years

Stage 2 – not stage 1 or 3

Stage 3 - High B2 (>5.5mg/L), survival <2 years]

Multiple Myeloma

How well did you know this?
1
Not at all
2
3
4
5
Perfectly
63
Q

What are the cytogenetics of multiple myeloma and the associated prognoses?

A

Cytogenetics:

Chrom 13 del (bad but good in CLL!),

Hypodiploidy (bad),

17p del (bad)

FISH: T(4:14) T(14;16)(bad) – by FISH only

T(11;14) ‘good’

How well did you know this?
1
Not at all
2
3
4
5
Perfectly
64
Q

Describe the sites of mutations and cell heritage for the following

What do they all have in common?

Chronic lymphocytic leukaemia

Acute lymphocytic leukaemia

Follicular lymphoma

Diffuse large b-cell lymphoma

Multiple myeloma

A

Chronic lymphocytic leukaemia- lymphoid tissue, mature naive b-cell

Acute lymphocytic leukaemia - bone marrow, pre-b-cell

Follicular lymphoma - lymphoid tissue, germinal centre b-cell

Diffuse large b-cell lymphoma - lymphoid tissue, germinal centre be cell

Multiple myeloma - plasma cell

Hodgkin lymphoma - lymphoid tissue, germinal centre b-cell

How well did you know this?
1
Not at all
2
3
4
5
Perfectly
65
Q

What are the unfavourable classifications of Hodgkin’s lymphoma?

A

Unfavorable Early Stage (B-Sx, age >50, ESR >50, bulky, 3+ areas)

Advanced Stage (III-IV)

How well did you know this?
1
Not at all
2
3
4
5
Perfectly
66
Q

What does ABVD stand for in the treatment of Hodgkin lymphoma?

What are the side effects?

A

Adriamycin

Bleomycin

Vinblastine

Dacarbazine

SEs

  • Hair loss
  • Neutropenia
  • Cardiotoxicity from adriamycin
  • Lung toxicity from bleomycin
  • Low risk of infertility
How well did you know this?
1
Not at all
2
3
4
5
Perfectly
67
Q

What does BEACOPP stand for in the treatment of Hodgkins lymphoma?

When is it used?

What are the drawbacks?

A

Bleomycin, Etoposide, Adriamycin, Cyclophosphomide, O-Vincristine, Procarbazine, Prednisone

Used for advanced Hodgkins or unfavourable stage

More risk of infertility and late-stage cancers

How well did you know this?
1
Not at all
2
3
4
5
Perfectly
68
Q

Why is low dose RTx used (where necessary) in Hodgkin’s lymphoma?

A

Reduces risk of late breast cancer in young women

How well did you know this?
1
Not at all
2
3
4
5
Perfectly
69
Q

What are the adverse prognostic factors in Hodgkin’s lymphoma? (Hasenclever Index)

What is the difference between having 1 or 5 or more of these?

A

Serum albumin < 40 g/L

Haemoglobin < 10.5 g/dL

Male gender

Stage IV disease

Total WCC > 15

Lymphocytes < 0.6, or < 8% of total WCC

Age > 45 years

Survival drops cumulatively - >5 = <50% PFS

How well did you know this?
1
Not at all
2
3
4
5
Perfectly
70
Q

Which chemotherapy drugs are associated with secondary CML?

A

Alkylators and etoposide

How well did you know this?
1
Not at all
2
3
4
5
Perfectly
71
Q

Aside from CML and breast cancer, what are the other cancers associated with Hodgkin’s treatment?

Which treatment do they stem from?

A

Thyroid, lung, bone - mainly XRT

Brain - unknown cause

How well did you know this?
1
Not at all
2
3
4
5
Perfectly
72
Q

What is the mechanism of secondary cardiovascular disease due to Hodgkins lymphoma treatment?

A

XRT

How well did you know this?
1
Not at all
2
3
4
5
Perfectly
73
Q

What are the survivorship issues in long term survivors of Hodgkin’s lymphoma?

A

Infertility - worse in men, worse if older

Second cancers - breast, lung, colon, leukaemia

Organ damage - heart (coronary disease), lung, thyroid

How well did you know this?
1
Not at all
2
3
4
5
Perfectly
74
Q

What is the significan of PET positive and PET negative disease in Hodgkin’s lymphoma?

A

FAR worse survival if PET positive

How well did you know this?
1
Not at all
2
3
4
5
Perfectly
75
Q

What is nodular lymphocyte predominant Hodgkin’s lymphoma?

Why is a misnomer?

How is it treated?

A

Unusual form of B-cell lympoma (Reed-Steenberg cells express CD20)

A misnomer due to the CD20 expression - it is a non-hodgkin’s lymphoma

May progress to DLBCL

Long survival if localised (surg +/- RTx)

Responds to anti-CD20 agent Rituximab - long survival interspersed with relapses

How well did you know this?
1
Not at all
2
3
4
5
Perfectly
76
Q

What are the commonest types of Non-Hodgkin lymphoma?

What are the other main types?

A

Follicular lymphoma

Diffuse large b-cell lymphoma

Others

Burkitt

SLL/CLL

MALT
Mantle cell

How well did you know this?
1
Not at all
2
3
4
5
Perfectly
77
Q

What are the cytogenetics of follicular non-Hodgkin lymphoma?

A

t(14;18) and bcl-2 rearrangement in > 85%

Follicular

  • t(14;18) —> overexpresson of BCL-2 —> failure of apoptosis (the usual mechanism of B-Cell death)
How well did you know this?
1
Not at all
2
3
4
5
Perfectly
78
Q

What is the epidemiology of follicular non-hodgkin lymphoma?

A

Median age at dx 60-65y

Male = Female

Western countries

How well did you know this?
1
Not at all
2
3
4
5
Perfectly
79
Q

What are the clinical features of follicular non-hodgkin lymphoma?

A

Generalised lymphadenopathy w/ BM involvement

+/- splenomegaly or hepatomegaly

Indolent clinical course

Cumulative risk of histologic transformation around 25%

B-symptoms in 10%

How well did you know this?
1
Not at all
2
3
4
5
Perfectly
80
Q

Should indolent stage IV non-hodgkin lymphoma be observed or treated? Why?

A

Treated with anti-CD20 antibodies (Rituximab)

Much better survival

How well did you know this?
1
Not at all
2
3
4
5
Perfectly
81
Q

What are the prognostic factors of follicular lymphoma? (Follicular lymphoma international prognostic index - FLIPI)

A

Age >60

Ann Arbor stage III/IV

Hb <120

LDH > ULN (Upper limit of normal)

>4 involved nodal sites

How well did you know this?
1
Not at all
2
3
4
5
Perfectly
82
Q

What is CHOP and R-CHOP?

When is it used?

A

Cyclophosphamide, H -Doxorubicin (Adriamycin), O-Vincristine, Prednisone

R- Rituximab

Used for Non-Hodgkin Lymphoma

How well did you know this?
1
Not at all
2
3
4
5
Perfectly
83
Q

What is the role of rituximab-maintenance therapy in NHL?

A

Improves event-free rate in comparison to observation alone.

Given for 2 years.

How well did you know this?
1
Not at all
2
3
4
5
Perfectly
84
Q

What is the potential role of lenalidomide in follicular lymphoma?

A

Augments rituximab, improves PFS - may replace CHOP in future.

How well did you know this?
1
Not at all
2
3
4
5
Perfectly
85
Q

What are the cytogenetics of DLBCL?

A

t(14;18) and bcl-2 rearrangement in < 30%

How well did you know this?
1
Not at all
2
3
4
5
Perfectly
86
Q

What is the epidemiology of DLBCL?

A

Median age at dx 55-60

Male >female

Minimal geographic variation

Around 50% disseminated at dx

How well did you know this?
1
Not at all
2
3
4
5
Perfectly
87
Q

What are the clinical features of DLBCL?

A

Usually

  • localised lymphadenopathy
  • BM involvement uncommon
  • B-sx in 30-40%
  • Aggressive but curable around 60% of cases
  • Risk of CNS involvement around 5%
How well did you know this?
1
Not at all
2
3
4
5
Perfectly
88
Q

When is the international prognostic index used?

What are it’s factors?

A

Predicts survival in NHL.

Performance status > 2

Age > 60 years

Ann Arbor stage III or IV

LDH above normal

2 extranodal sites of disease

How well did you know this?
1
Not at all
2
3
4
5
Perfectly
89
Q

Describe the ECOG score

A

0 – Asymptomatic

1 – Symptomatic but completely ambulatory (Restricted in physically strenuous activity but ambulatory. ie light housework, office work)

2 – Symptomatic, <50% in bed during the day (Ambulatory and capable of all self care but unable to carry out any work activities. Up and about more than 50% of waking hours)

3 – Symptomatic, >50% in bed, but not bedbound (Capable of only limited self-care, confined to bed or chair 50% or more of waking hours)

4 – Bedbound (Completely disabled. Cannot carry on any self-care. Totally confined to bed or chair)

5 – Death

How well did you know this?
1
Not at all
2
3
4
5
Perfectly
90
Q

What two prognostic subgroups have been identified by DNA microarray analysis in DLBCL? Which one has better survival?

A

Germinal-center B-Cell like (better survival)

Type 3 (worse)

Activated B-cell like (worst)

How well did you know this?
1
Not at all
2
3
4
5
Perfectly
91
Q

What is first line therapy for DLBCL?

A

R-CHOP

How well did you know this?
1
Not at all
2
3
4
5
Perfectly
92
Q

What are the most significant risk factors for CNS disease in lymphoma?

What are the independent risk factors?

A
  • testicular involvement
  • clinical stage
  • IPI score

Independent

  • elevated LDH
  • >1 extranodal disease site
  • ECOG >2
How well did you know this?
1
Not at all
2
3
4
5
Perfectly
93
Q

When is intrathethecal methotrexate used? Why is concurrent IV used?

What is the role of rituximab?

A

Used in DLBCL if risk factors for CNS disease present (extranodal site + 1 other risk factor i.e. LDG, ecog, testicular)

Intrathecal does not get into brain parenchyma, high dose MTx crosses the BBB.

Reduces CNS disease in patients with IPI of 0 or 1

How well did you know this?
1
Not at all
2
3
4
5
Perfectly
94
Q

What is a ‘double hit’ lymphoma?

What is it’s relevance?

A

Dual positivity for BCL-2 + C-MYC portends a poor prognosis, either de novo or with t-FL

No standard treatment

Rituximab-containing therapy provides a benefit but is still inadequate

How well did you know this?
1
Not at all
2
3
4
5
Perfectly
95
Q

What are the cytogenetics of Burkitt’s NHL?

A

Burkitt’s NHL: t(8;14) & c-myc, highly aggressive, curable

  • t(8;14) —> overexpreesion of c-myc —> malignant transformation through excess b-cell proliferation
  • associated with abdominal pain/fullness as disease prefers abdomen
How well did you know this?
1
Not at all
2
3
4
5
Perfectly
96
Q

What is associated with primary cerebral NHL? What is the standard treatment?

A

HIV-related or sporadic

Standard is high dost MTx and XRT

How well did you know this?
1
Not at all
2
3
4
5
Perfectly
97
Q

What viruses are associated with HIV-related NHL?

A

HHV-8, EBV, also related to CD4 count

Often extra-nodal, aggressive.

How well did you know this?
1
Not at all
2
3
4
5
Perfectly
98
Q

What are the cytogenetics and location of disease in mantle-cell NHL?

A
  • t(11;14) and bcl-1 translocation (cyclin D1)
  • frequent GI involvement (lymphomatous polyposis)
  • aggressive clinical behaviour, treatment difficult
How well did you know this?
1
Not at all
2
3
4
5
Perfectly
99
Q

What genetic mutation is associated with 100% of hairy-cell leukaemias?

A

B-RAF

How well did you know this?
1
Not at all
2
3
4
5
Perfectly
100
Q

What does overexpression of BCL-2 result in, in the context of lymphoma?

A

resistance to apoptosis

How well did you know this?
1
Not at all
2
3
4
5
Perfectly
101
Q

What is Bruton’s tyrosine kinase?

When is an anti-BTK drug useful?

A

BTK is a kinase that plays a crucial role in B-cell development.

Ibrutinib Is Highly Effective in Relapsed /Refractory CLL

How well did you know this?
1
Not at all
2
3
4
5
Perfectly
102
Q

What are the marginal zone lymphomas?

A

MALT type

Nodal type

Splenic type

How well did you know this?
1
Not at all
2
3
4
5
Perfectly
103
Q

Name the indolent lymphomas

A
  • follicular, marginal zone, SLL/CLL
How well did you know this?
1
Not at all
2
3
4
5
Perfectly
104
Q

What is the treatment for MALT lymphoma?

A

MALT - H-pylori and PPIs w/ frequent endoscopy, or whole-stomach XRT

How well did you know this?
1
Not at all
2
3
4
5
Perfectly
105
Q

Name the aggressive lymphomas

A

DLBCL, Mantle cell lymphoma, primary CNS lymphoma, T-Cell lymphoma, high grade (Burkitt or lymphoblastic)

How well did you know this?
1
Not at all
2
3
4
5
Perfectly
106
Q

What is the treatment for DLBCL and mantle-cell lymphoma?

A

R-CHOP

Maybe Auto-SCT

How well did you know this?
1
Not at all
2
3
4
5
Perfectly
107
Q

What is the treatment for primary CNS lymphoma?

A

Repetitive cycles of high-dose intravenous methotrexate with rituximab early in the treatment course produce better results than whole brain radiotherapy and with less cogni- tive impairment.

How well did you know this?
1
Not at all
2
3
4
5
Perfectly
108
Q

What is the treatment for high grade lymphomas? (Burkitt or Lymphoblastic)

A

Intense, cyclic chemotherapy in the hospital similar to that given for ALL, and they also require intrathecal chemotherapy as central nervous system prophylaxis.

How well did you know this?
1
Not at all
2
3
4
5
Perfectly
109
Q

How do peripheral t-cell lymphomas fare compared to B-cell disease?

A

Peripheral T-cell lymphomas usually have advanced stage nodal and extranodal disease and typically have inferior response rates to therapy compared to patients with aggressive B-cell disease

How well did you know this?
1
Not at all
2
3
4
5
Perfectly
110
Q

What is the prognosis of indolent non-hodgkin lymphoma?

A

10–15 years. These diseases ultimately become refractory to chemotherapy.

How well did you know this?
1
Not at all
2
3
4
5
Perfectly
111
Q

What is the likelihood of survival after relapse of non-hodgkins lymphoma?

A

Depends on whether the lymphoma is still responsive to chemotherapy. If the lymphoma remains responsive to chemotherapy, autologous hematopoietic stem cell transplantation offers a 50% chance of long-term lymphoma-free survival.

How well did you know this?
1
Not at all
2
3
4
5
Perfectly
112
Q

Describe the Ann-Arbor staging system.

What is it used for?

A

Used for Hodgkin’s lymphoma.

Ann Arbor

I - one LN

II - 2 or more on one side of diaphragm

III - Both sides of diaphragm

IV - disseminated w/ extranodal involvement

A - no constitutinal sx

B - 10% wt loss of six months, fever, drenching night sweats (one or more)

How well did you know this?
1
Not at all
2
3
4
5
Perfectly
113
Q

What are the clinical findings of Hodgkin Lymphoma?

A

Bimodal - age 20s and then age 50s

Painless mass, commonly in the neck

Constitutional symptoms

Pain in lymph node following alcohol ingestion

Starts in one lymph node then spreads contiguously

How well did you know this?
1
Not at all
2
3
4
5
Perfectly
114
Q

What is the treatment for Hodgkin lymphoma?

A

Treatment

  • RTx for stage 1A with high cervical LN and low ESR

Otherwise - ABVD (BEACOPP if bad but toxic + infertility and no definite survival advantage)

Stage I-II - usually short course ABVD +/- RTx

  • full course if bulky disease

II-IV - full course ABVD w/ no RTx

Must watch for bleomycin plum toxicity.

Relapse may still be curable with CTx - high dose + auto SCT - 35-50% chance of cure.

How well did you know this?
1
Not at all
2
3
4
5
Perfectly
115
Q

What is the prognosis of Hodgkin lymphoma?

A

Prognosis

Factors: Stage, age, gender, Hb, alb, abc and lymphocyte count

Cure rate 75% if zero to two factors present, excess of 90% if Ia or IIa disease.

How well did you know this?
1
Not at all
2
3
4
5
Perfectly
116
Q

What is the pathogenesis of non hodgkin lymphoma?

A

Oncogene juxtaposed next to a either an Ig-gene (B-cell lymphoma) or t-cell receptor or related gene (t-cell lymphoma) —> painless lymphadenopathy —> indolent to rapidly progressive.

How well did you know this?
1
Not at all
2
3
4
5
Perfectly
117
Q

What are the prominent non-hodgkin lymphomas?

A

85% of NHL are B-cell, mainly DLBCL and follicular

How well did you know this?
1
Not at all
2
3
4
5
Perfectly
118
Q

What are the lab findings in non-hodgkin’s lymphoma?

A

Normal peripheral blood, no circulating lymphoma cells

Elevated LDH (prognostic)

BMB - paratrabecular monoclonal lymphoid aggregates

LP - (high grade) - meningeal involvement w/ malignant cytology

CXR - mediastinal mass

How well did you know this?
1
Not at all
2
3
4
5
Perfectly
119
Q

How do you make the diagnosis of non-hodgkin’s lymphoma?

A

Tissue bx (LN or extra nodal tissue) for dx and classification

How well did you know this?
1
Not at all
2
3
4
5
Perfectly
120
Q

What is the treatment for the indolent non-hodgkin lymphomas?

A

Indolent (follicular, marginal zone, SLL/CLL)

Limited disease (1-2 contiguous LNs) —> XRT for cure

Most are disseminated though.

  • R-CHOP or R-CVP
  • allo-SCT if aggressive and low grade

MALT - H-pylori and PPIs w/ frequent endoscopy, or whole-stomach XRT

How well did you know this?
1
Not at all
2
3
4
5
Perfectly
121
Q

What is the treatment for the aggressive non-hodgkin lymphomas?

A

DLBCL - RCHOP +/-Auto SCT if responsive to chemo

Mantle cell - as above

Primary CNS - high dose IV MTx + rituximab

High grade lymphoma (Burkitt/Lymphoblastic) - intense CTx plus intrathecal CTx as prophylaxis

Peripheral T-cell lymphomas - poor response, usually disseminated disease.

How well did you know this?
1
Not at all
2
3
4
5
Perfectly
122
Q

What is the prognosis for the indolent lymphomas?

A

Indolent - 10-15 years, ultimately becomes refractory and progresses

How well did you know this?
1
Not at all
2
3
4
5
Perfectly
123
Q

What is the IPI (International Prognostic Index) used for in regards to non-hodgkin lymphoma?

What are the factors?

A

IPI (International Prognostic Index) stratifies risk of those with intermediate grade lymphoma

factors:

Age >60,

elevated LDH,

Stage III, or IV disease.

80% chance of complete response rate with no or one risk factors

How well did you know this?
1
Not at all
2
3
4
5
Perfectly
124
Q

What are the two critical histopathological features of Hodgkin lymphoma?

A

Reed-Sternberg cells in an appropriately reactive (inflammatory) cellular background

How well did you know this?
1
Not at all
2
3
4
5
Perfectly
125
Q

What are the clinical findings of Hodgkin lymphoma?

A

Bimodal - age 20s and then age 50s

Painless mass, commonly in the neck

Constitutional symptoms

Pain in lymph node following alcohol ingestion

Starts in one lymph node then spreads contiguously

How well did you know this?
1
Not at all
2
3
4
5
Perfectly
126
Q

What are the prognostic factors for Hodgkin lymphoma?

A

Prognosis

Factors: Stage, age, gender, Hb, alb, WBC and lymphocyte count

Cure rate 75% if zero to two factors present, excess of 90% if Ia or IIa disease.

How well did you know this?
1
Not at all
2
3
4
5
Perfectly
127
Q

What is the aetiology of the acute leukaemias?

A

Aetiology

  • AML: adult disease around 60, increased incidence with age
  • ALL: 20% of adult acute leukaemia’s
  • no clear cause
  • radiation, benzene
  • prior Ctx - cyclophos, melphalan, other alkylators, etoposide
  • after toxins/CTx —> MDS w/ Cr 5 & 7 abnormalities; etoposide Cr 11q23
How well did you know this?
1
Not at all
2
3
4
5
Perfectly
128
Q

What is APL? Why does it differ from the other acute leukaemias?

A

APL (Acute promyelocytic leukaemia)

  • t(15;17) —> PML-RARa fusion gene -> interacts w/ retinoid acid receptor causing differentiation block.
  • can give ATRA —> differentiated into neutrophils occur —> neutrophils apoptose within minutes —> cure!
How well did you know this?
1
Not at all
2
3
4
5
Perfectly
129
Q

What are the favourable cytogenetics in AML?

A

Favourable - ‘core binding factors’ 15% of cases

  • t(8;21)
  • inv(16)
  • p(13;q22)
How well did you know this?
1
Not at all
2
3
4
5
Perfectly
130
Q

What are the unfavourable cytogenetics in AML?

A

Unfavourable - poor prognosis

-5, -7, del(5q), Abnormal 3q, Complex cytogenetics

2 or more monosomy’s

3 or more separate cytogenetic abnormalities

How well did you know this?
1
Not at all
2
3
4
5
Perfectly
131
Q

What is the significance of the intermediate-risk subgroup of AML?

A
  • normal cytogenetics or abnormalities w/o strong prognostic significance
  • new molecular markers
  • FLT3-ITD (internal tandem duplication)
  • 30% of CN-AML (cytogenetically normal AML)
  • very poor prognosis
  • NPM1 (nucleophospmin 1) - favorable prognosis
How well did you know this?
1
Not at all
2
3
4
5
Perfectly
132
Q

What are the molecular markers of AML and what are their prognoses?

A
  • FLT3-ITD (internal tandem duplication)
  • 30% of CN-AML (cytogenetically normal AML)
  • very poor prognosis
  • NPM1 (nucleophospmin 1) - favorable prognosis
How well did you know this?
1
Not at all
2
3
4
5
Perfectly
133
Q

What are the cytogenetics of ALL?

A
  • Common, early B, or T-cell

Hyperdiploidy has a better prognosis but is in kids

Unfavourable: Philadelphia chromosome and t(4;11) which has fusion genes involving the MLL gene at 11q23

How well did you know this?
1
Not at all
2
3
4
5
Perfectly
134
Q

What are the clinical findings of ALL?

A

Clinical findings

  • presentation of days to weeks
  • bleeding of skin and mucosal surfaces due to thrombocytopaenia
  • DIC (APL and monocytic leukaemia)
  • Infection due to neutropaenia; gram negative bacteria (e.coli, Klebsiella, pseudomonas), or fungi (candida, aspergillus —> cellulitis, pneumonia, perirectal infections.
  • Gum hypertrophy, bone and joint pain
  • Hyperleukocytosis - impaired circulation —> headache, confusion, dyspnoea, need urgent leukopheresis and chemotherapy - 40% mortality
How well did you know this?
1
Not at all
2
3
4
5
Perfectly
135
Q

What are the examination findings in ALL?

A

Pale, purpura, petechiae

Stomatits, gum hypertrophy and rectal fissues in monocytic leukaemia

Hepatosplengomegaly, lymphadenopathy

Bone tenderness: sternum, tibia, femur

How well did you know this?
1
Not at all
2
3
4
5
Perfectly
136
Q

What are the lab findings in ALL?

A

Hallmark: Pancytopaenia with circulating blasts

Bone marrow: Hypercellular, >20% blasts

Hyperuricaemia

DIC: low fibrinogen, fibrin degradations products

ALL: Mediastinal mass on CXR

Meningeal leukaemia: Blasts in spinal fluid - 5% of cases, mainly monocytic AML

Myeloperoxidase stain: Auer rod - eosinophilic needle-like inclusion in the cytoplasm = pathognomonic of AML (but has been superceded by flow cytometry)

(Not in ALL because peroxidase is only in myeloid cells)

How well did you know this?
1
Not at all
2
3
4
5
Perfectly
137
Q

Describe the flow cytometry findings of AML and ALL.

A

Flow cytometry:

AML - CD13 or CD33 (myeloid antigens)

ALL - B-lineage: CD19 (all B-Cells), and CD10 (common ALL antigen)

T-lineage - CD2, 5, and 7 (some combo of), but NOT mature t-cell markers, i.e. 3, 4, or 8, and not surface iG.

Almost all ALL cells express terminal deoxynucleotidyl transferase (TdT) (but not Burkitt type ALL)

How well did you know this?
1
Not at all
2
3
4
5
Perfectly
138
Q

What is the treatment of AML?

A
  • Anthracycline (daunorubicin or idarubicin) plus cytarabine - complete remission in 80-90% under 60, 50-60% older patients
  • Intermediate risk - cure rates ascend from 35-40% w/ CTx to 50-60% with allo-transplant
  • Positive core binding factor cytogenetics have a more favourable prognosis with CTx, while CTx alone is not helpful if unfavourable profile.
  • FLT3 inhibitors are in trial.

Unfavourable - allo-transplant but poor cure rates 20-30%, worse if over 60 even if in first remission - may be a role for reduced intensity allo-transplant.

  • Prognosis drops with recurrence after first remission.
How well did you know this?
1
Not at all
2
3
4
5
Perfectly
139
Q

What is the treatment of APL?

A
  • Anthracycline plus ATRA - 90-95% CR
  • High risk (higher WBC count) - added arsenic improves outcomes
  • Arsenic can produce second remission in 90% of cases if required.
How well did you know this?
1
Not at all
2
3
4
5
Perfectly
140
Q

What is the treatment of ALL? What if they are Ph+?

A
  • Vincristine, Daunorubicin, Asparaginase, PreD (VDAC)
  • CR in 90%
  • Ph+ needs TKI in addition to standard CTx as T315i (resistant to all 3 TKIs) mutation much more common than in CML, unless older - 90% can achieve remission
  • Need CNS prophylaxis to avoid meningeal sequestration of leukaemia cells.
  • Low risk: 70% chance of cure with
  • Intermediate: 30-50% chance
  • High risk (adverse cytogenetics or poor response to CTx) —> Allo-BMT
How well did you know this?
1
Not at all
2
3
4
5
Perfectly
141
Q

What is the prognosis in AML?

A

AML: High dose post remission CTx —> cure in 35-40%, high dose cytarabine superior

Allo BMT curative in 50-60% of cases

Cure rates for older patients very low even with remission

How well did you know this?
1
Not at all
2
3
4
5
Perfectly
142
Q

How is complete remission (CR)

in AML defined?

A

<5% blasts on bmbx

How well did you know this?
1
Not at all
2
3
4
5
Perfectly
143
Q

Which leukaemia is a widened mediastinum a hallmark of?

A

ALL

How well did you know this?
1
Not at all
2
3
4
5
Perfectly
144
Q

Question 5

In females under 20 years of age treated successfully with radiotherapy for Hodgkin’s disease, which one of the following is the most frequent second tumour occurring after 10 years?

A. Acute leukaemia.

B. Thyroid cancer.

C. Non-Hodgkin’s lymphoma.

D. Breast cancer.

E. Ovarian carcinoma.

A

D

How well did you know this?
1
Not at all
2
3
4
5
Perfectly
145
Q

A 32-year-old man presents with weight loss, splenomegaly 20 cm below the left costal margin and a white cell count of 480 x 109/L [3.5-9.5]. Bone marrow examination confirms the diagnosis of an accelerated phase myeloid leukaemia in chronic phase.

Which treatment option offers this man the best chance of long-term disease-free survival?

A. Imatinib.

B. Interferon and cytarabine.

C. Hydroxyurea.

D. Autologous bone marrow transplantation.

E. Allogenic stem cell transplantation.

A

A.

Patients with advanced-stage disease (accelerated phase or myeloid/lymphoid blast crisis) should be treated with a tyrosine kinase inhibitor alone or in combination with myelosuppressive chemotherapy. The doses of tyrosine kinase inhibitors in that setting are usually higher than those appropriate for chronic-phase disease. Since the duration of response to tyrosine kinase inhibitors in this setting is limited, these patients should ultimately be con- sidered for allogeneic stem cell transplantation.

How well did you know this?
1
Not at all
2
3
4
5
Perfectly
146
Q

A 62-year-old woman underwent an elective cholecystectomy. Routine full blood examination revealed:

Physical examination revealed no lymphadenopathy and no hepatosplenomegaly. The woman was asymptomatic. Flow cytometry revealed that the lymphocytosis marked as CD5+, CD19+ and CD23+.

What is the most appropriate management of this patient?

A. Observation alone.

B. Prednis(ol)one.

C. Chlorambucil.

D. Combination chemotherapy.

E. Autologous bone marrow transplantation.

A

Answer - A

Treatment

Early stage - observation

Indications for tx: progressive fatigue, symptomatic lymphadenopathy, anaemia or thrombocytopaenia (either symptomatic and progressive Rai II, or stage III/IV disease)

Staging - Rai

Stage 0 - lymphocytosis (low-risk)

Stage II - organomegaly (low risk)

Stage III - anaemia (high risk)

Stage IV - thrombocytopaenia (high risk)

How well did you know this?
1
Not at all
2
3
4
5
Perfectly
147
Q

Question 53

A patient bleeds during low molecular weight heparin treatment.

Which one of the following best indicates whether the heparin is contributing to bleeding?

A. The activated partial thromboplastin time.

B. The thrombin clotting time.

C. The antifactor Xa activity.

D. The prothrombin time.

E. The tissue factor inhibitor assay.

A

Answer: C

148
Q

Describe the second test you would do after confirming a prolonged APTT and the meaning of it’s results.

A

Mixing study: 50% pooled:50% pt plasma

Imm/Incubated Correction = factor deficiency

Imm/Incubated delay: Inhibitor (heparin)

Immediate correction/Incubated delay = delayed inhibitor present

Inhibitors: Heparin, Lupus AC (in vivo), acquired factor inhibitors (commonly VIII)

Deficiencies: VIII (a), IX (b), VWF (VIII carrier)

149
Q

Which cytotoxics cause peripheral neuropathy?

A

Platins

Vincristine

150
Q

What are the indications, MOA, and main side effect of hydroxyurea?

A

Polycythaemia vera and essential thrombocytosis

Inhibits ribonucleotide reductase, decreasing DNA synthesis

Myelosuppression

151
Q

What side effect does the antimetabolite drugs all have in common?

A

Myelosuppression

152
Q

What is Haemophilia A?

What are it’s clinical findings?

How is it diagnosed?

How is it treated?

A

Hamophilia A (VIII Deficiency)

  • X-linked recessive (men get, women carry)
  • 30% are spontaneously ACQUIRED
  • less common than B, more severe
  • mild >5% (no bleed) /mod 1-5% (variable) / severe <1% (VIII deficiency)
  • Once bleeding, all phenotypes bleed the same
  • Haemarthroses (knee>elbow>ankle) 70-80%
  • CNS bleeds in severe phenotype
  • Tx: prophylaxis in severe (aim to reduce to mod), early tx of bleeds with factor replacement (at home if simple), avoid antiplatelets, regular exercise, replace factors (plasma/recombinant, cry in developing world), DDAVP (increases VIII and VWB levels, transexamic acid (antifibrinolytics) to stop clot breakdown
153
Q

What is vonWillebrands disease?

A

VWB (deficiency or dysfunction

  • multimeric protein, plasma & endothelial cells (Weibel-palade bodies)
  • Expressed on activated endothelial cells, released from alpha granules of activated platelets
  • synthesized in megakaryocytes
  • T ½ = 12 hours
  • binds plt at injury site, carries VIII
  • decreased plt adhesion, decreased VIII → bleeding
  • Increased by: age, African, non O blood, inotropes (aderenaline), inflammation, hormones (OCP, pregnancy, periods)
  • Type 1: partial quantitative deficiency, decreased VWF levels, likely gene mutation, significant bleeding and FHx of.
  • Type 3: Rare, near complete deficiency

Tx: DDAVP (increases VIII and VWB levels), human derived VWF with VIII, antifibrinolytics (transexamic acid)

154
Q

Which is more common, haemophilia A or B?

A

B. Factor 9 deficiency

155
Q

What are the causes of a prolonged APTT and INR?

A

Global dysfunction (APTT & INR)

DIC = plus low fibrinogen and high d-dimer

Liver disease → synthetic ftn (check albumin), TV factors
Direct thrombin inhibitors / Xa inhibitors (dabigatran and rivaroxaban)
Common pathway deficiency (rare)

156
Q

What are the lab findings in DIC?

A

DIC = Low APTT and INR plus low fibrinogen and high d-dimer

157
Q

What does the INR/PT look at in terms of the coagulation cascade? What prolongs it?

A

INR/PT (Extrinsic/Tissue factor pathway)

Fibrinogen, prothrombin, V, VII, & X

Particularly sensitive to VII

Prolonged by Vit-K antagonism (Warfarin), deficiency (ETOH/malabsorption), undx VII deficiency

Relationship to factor-level is non-linear (still have 30% of factors if level is 2, low bleeding risk)

Curve plateaus at 3 and beyond so bleeding risk same between 3-14

This is the main pathway activated by endothelial damage

158
Q

Which coagulation pathway is activated by endothelial damage?

A

Extrinsic pathway

159
Q

What does the anti-Xa level test?

A

Anti-Xa assay

Measures heparin effect

Can use to titrate dose down

160
Q

What is the Thrombin Clotting Time (TCT) test used for?

A

Thrombin Clotting Time (TCT)

Used when mixing study not corrected - looks for an inhibitor

Increased by heparin (reptilase normal), faulty fibrinogen, increased fibrin degradation products, and dabigatran

161
Q

What to protein C & S do?

A

Protein C & S (anticoagulants)

Activated by thrombin to form a negative feedback loop. Forms the APC (activated protein C) complex with protein S, which then degrades Factor V and VIIIa (so blocks the common pathway and last step of the intrinsic pathway respectively

162
Q

What does the reptilase test do?

A

Reptilase test - looks for an acquired inhibitor that isn’t heparin

Reptilase splits fibrinogen →fibrin

Not prolonged by heparin

163
Q

Which factors belong to the intrinsic coagulation pathway?

A

12, 11, 9, and 8

164
Q

Which factors belong to the extrinsic coagulation pathway?

A

7 and tissue factor

165
Q

Which factors belong to the common coagulation pathway?

A

Factors 10, 5, 2 (prothrombin), and 1 (fibrinogen)

166
Q

What does factor 12 do?

A

Starts the intrinsic pathway coagulation cascade

167
Q

What does factor VII do?

A

The main role of factor VII (FVII) is to initiate the process of coagulation in conjunction with tissue factor (TF/factor III). Tissue factor is found on the outside of blood vessels - normally not exposed to the bloodstream.

168
Q

What does factor VIII do?

A

In response to injury, coagulation factor VIII is activated and separates from von Willebrand factor.

169
Q

What is the main role of the extrinsic pathway?

A

The main role of the tissue factor pathway is to generate a “thrombin burst”, a process by which thrombin, the most important constituent of the coagulation cascade in terms of its feedback activation roles, is released very rapidly.

170
Q

What is the role of factor V?

A

Factor V: Procoagulant that amplifies thrombin production. Small amount of thrombin at a wound activates factor V via limited proteolysis, which in turn becomes a cofactor in the prothrombinase complex, cleaving prothrombin to generate more thrombin in a positive feedback loop.

Thrombin then generates a negative feedback loop by converted protein C to activated protein C.

171
Q

What is the role of protein C in the coagulation cascade?

A

Activated protein C (APC): natural anticoagulant, cleaves and inactivates activated factor V and VIII which then reduces thrombin production.

APC affects both factor V and factor Va (activated V) to increase clotting risk.

  1. When APC cleaves factor V (the inactive version), it (factor V) becomes a cofactor in further degration of Va and VIIIa. With the point mutation at position 506 on Factor V, APC can’t cleave the factor, it becomes less effective as a cofactor and the degradation
  2. APC performs a series of sequential cleavages on Factor Va, the first of which is the position 506 point. With this site removed, sequential cleavage cannot happen, resulting in 20x slower degration of the factor (factor V Leiden). With the activated factor unable to be cleaved, more thrombin continues to be generated.

These effects contribute equally to thrombosis and explain the stronger presentation in homozygotes.

172
Q

What is leucodepletion? Why is it important?

A

Leucodepletion is the removal of white blood cells from a blood component. In Australia, this applies to whole blood, red cells and platelets.

The potential benefits of leucodepletion for patients include:

  • Reduction in platelet refractoriness


- Reduction in febrile non-haemolytic transfusion reactions. (FNHTR)


- Reduction in CMV transmission risk


- Improved chance of finding an organ transplant match if required


- Reduction in storage lesion effect


173
Q

Which of the following transfusion reactions is leucodepletion most effective in preventing?

A. Febrile non-haemolytic transfusion reaction. 


B. Transfusion associated graft versus host disease. 


C. Anaphylaxis.


D. Transfusion related acute lung injury. 


E. Delayed haemolytic transfusion reaction.

A

Answer: A

174
Q

What is a febrile non-haemolytic transfusion reaction?

What causes it?

A
  1. Unexpected temperature rise (≥38ºC or ≥1ºC above baseline, if baseline ≥37ºC) during or shortly after transfusion. Usually an isolated finding.
  2. Caused by leucocyte antigen-antibody reaction and by cytokine accumulation produced by leucocytes in transfused blood. Formation of cytokines can be avoided if the leucocytes can be removed immediately (leucodepletion)

FNHTR is also caused by the presence of recipient antibodies (raised as a result of previous transfusions or pregnancies) reacting to donor human leucocyte antigens (HLA) or other antigens. These antigens are present on donor lymphocytes, granulocytes, or platelets.

175
Q

With regards to blood products, what is a storage lesion?

How can you reduce it?

A

The changes occurring in storage of RBCs including increased ammonium, free Hb, and potassium, and decreased ATP, 2,3DPG, labile proteins (complement, coagulation factors, fibronectin), sodium, and pH.

Leucocyte degradation during storage results in cell fragments which may not be removed by post-storage filtration - can provoke HLA or platelet alloimmunisation. In leucodepleted blood - there is a lower incidence of alloimmunisation and (possibly) diminished immunomodulation that may result from the transfusion of membrane fragments.

176
Q

What is the most common cause of transfusion-related fatalities?

What are the symptoms, timeframe, and incidence?

A

TRALI


Symptoms: Acute onset of fever, chills, dyspnoea, tachypnoea, tachycardia, hypotension, hypoxaemia and noncardiogenic bilateral pulmonary oedema leading to respiratory failure during or within 6 hours of transfusion.

Implicated in transfusion of unfractionated plasma-containing components.


Incidence: 1:10,000 most commonly reported.

177
Q

With regard to transfusion, what constitiutes a delayed haemolytic transfusion reaction?

Which blood groups are associated with it?

How do you treat it?

A


Patients may present with unexplained fever and anaemia usually 2 to 14 days after transfusion of a red cell component.

Unexplained fever and anaemia 2-14 days after transfusion of red-cell component. May also have jaundice, high bilirubin, high LDH, reticulocytosis, spherocytosis, positive antibody screen and a positive Direct Antiglobulin Test (DAT).

Causes: After transfusion, transplantation or pregnancy, a patient may make an antibody to a red cell antigen that they lack. If the patient is later exposed to a red cell transfusion which expresses this antigen a DHTR may occur. DHTRs may also occur with transfusion transmitted malaria and babesiosis.

Blood group antibodies associated with DHTRs include those of the Kidd, Duffy, Kell and MNS systems, in order of decreasing frequency.

Most delayed haemolytic reactions have a benign course and require no treatment, however life-threatening haemolysis with severe anaemia and renal failure may occur.

If an antibody is identified, you may request for antigen-negative blood if further transfusion is needed.

178
Q

Name the thrombin inhibitors

A

Dabigatran

Argotroban

Bivalriduin

Lepirudin

179
Q

Name the XA inhibitors)

A

Xa Inhibitors

Rivaroxaban

Apixaban

Fondaparinux (via AT

Danaprinoid

180
Q

Name the combined Xa & thrombin inhibitors

A

Heparin

181
Q

Name the vitamin K antagonist and what it inhibits

A

Warfarin (Thrombin (II), VII, IX and X) via impaired hepatic synthesis.

182
Q

What is the mechanism of action, clearance and indications for Rivaroxaban? What blood tests does it affect?

A

Factor Xa inhibitor

T ½ = 12-17h Peak 1-3h

Renally excreted

Minimally protein bound

Irreversible

Monitor: dilute thrombin time (special)

Prolongs APTT & INR

Does not affect fibrinogen level

Indicated for stroke prevention (non-valvular AF), thromboprophylaxis post hip and knee replacement

183
Q

How do you manage bleeding associated with dabigatran and rivaroxaban?

A

Dabigatran or Rivaroxaban Bleeding

Coag screen, G&H, usual bloods, stop drug

Mild: compression, antifibrinolytics (tranexamic acid) QID, delay further oral anticoagulation

Mod/Severe: Antifibrinolytics, as for usual bleeding, transfuse, consider charcoal if <2h for dabigatran, <8h for rivaroxaban

Consider HD for dabigatran

184
Q

How does dabigatran work? What blood tests does it affect? Indications?

How do you manage bleeding?

A

Direct thrombin (IIa) inhibitor

T ½ = 7-11h Peak 1-3h

50% renally excreted

Highly protein bound

Irreversible

Prolongs APTT & INR, TCT

Does not affect fibrinogen level

Indicated for stroke prevention (non-valvular AF), thromboprophylaxis post hip and knee replacement, treatment of DVT

Bleeding: INR, APTT,TCT, Fibrinogen, check renal ftn, if TCT normal then low dabigatran level, supportive, tranexamic acid, charcoal, dialysis

185
Q

What test would you do to see if the dabigatran level was low?

A

Thrombin clotting time. It will be normal if the dabigatran level is low.

186
Q

How does heparin work?

A

Heparin binds to the enzyme inhibitor antithrombin III (AT), causing a conformational change that results in its activation through an increase in the flexibility of its reactive site loop. The activated AT then inactivates thrombin and other proteases involved in blood clotting, most notably factor Xa.

187
Q

What is tranexamic acid? How does it work? When is it useful?

A

It is an antifibrinolytic that competitively inhibits the activation of plasminogen to plasmin, by binding to specific sites of both plasminogen and plasmin, a molecule responsible for the degradation of fibrin, a protein that forms the framework of blood clots.

Useful where bleeding is going to be a problem, i.e dental extraction, certain surgeries.

188
Q

Describe factor V leiden disease:

  • mechanism
  • epidemiology
  • clinical findings
  • when to test
A

Factor V Leiden

  • mutant Factor V, can’t be switched off by APC so clotting continues.
  • most common hereditary hyper coagulability disorder among caucasians.
  • restricted to veins (so DVT and PE, not strokes or heart attaches)
  • happens in 30% of unprovoked DVT/PE
  • risk increases in pregnancy or oestrogen containing drugs.
  • suspect in any DVT patient under the age of 45
189
Q

How do you test for factor V leiden disease?

A

Most laboratories screen ‘at risk’ patients with either a snake venom (e.g. dilute Russell’s viper venom time) based test or an aPTT based test. In both methods, the time it takes for blood to clot is decreased in the presence of the factor V Leiden mutation. This is done by running two tests simultaneously; one test is run in the presence of activated protein C (APC) and the other, in the absence. A ratio is determined based on the two tests and the results signify to the laboratory whether APC is working or not. These are quick, three minute, automated tests that most hospital laboratories can easily perform.

190
Q

A 42-year-old previously healthy woman presents with an above-knee deep venous thrombosis of the right leg complicated by probable pulmonary thrombo-embolism. She has no clinical risk factors such as recent surgery or trauma.

In the investigation of her thrombo-embolic disease, which one of the following is the most likely abnormality to be found?

A. Lupus anticoagulant.

B. Protein C deficiency.

C. Protein S deficiency.

D. Dysfibrinogenaemia.

E. Factor V Leiden.

A

Answer E. The most common of the inherited disorders of DVT.

191
Q

What does the prothrombin time measure?

A

PROTHROMBIN TIME (PT) — The prothrombin time (PT) (figure 1) is used to assess the extrinsic pathway of clotting, which consists of tissue factor and factor VII, and coagulation factors in the common pathway (factors II [prothrombin], V, X, and fibrinogen).

192
Q

What does the aptt measure?

A

ACTIVATED PARTIAL THROMBOPLASTIN TIME (aPTT) — The activated partial thromboplastin time (aPTT or PTT) is used to assess the integrity of the intrinsic coagulation pathway (prekallikrein, high molecular weight kininogen, factors XII, XI, IX, VIII) and final common pathway (factors II, V, X, and fibrinogen), and to monitor therapy with unfractionated heparin.

193
Q

What does FFP contain and when is it used?

A

Fresh frozen plasma comprises the acellular component of blood and contains all of the coagulation factors. It is used in patients with coagulopathy who are bleeding, or at risk of bleeding, when more specific therapy is not appropriate or available. Fresh frozen plasma is most commonly used in massive transfusion, cardiac bypass, liver disease or acute disseminated intravascular coagulopathy.

It is essential as the exchange product in plasmapheresis for TTP

194
Q

What is in cryoprecipitate and when is it used?

A

Mainly used in DIC to replace fibrinogen. Sometimes to correct platelet dysfunction in CKD.

Derived from FFP.

Cryoprecipitate contains most of the factor VIII, factor XIII, von Willebrand factor, and fibrinogen.

Cryodepleted plasma can be used in plasma exchange for thrombotic thrombocytopenic purpura.

195
Q

What does Factor XIII do?

A

Factor XIII or fibrin stabilizing factor is an enzyme (EC 2.3.2.13) of the blood coagulation system that crosslinks fibrin. Upon activation by thrombin, factor XIIIa acts on fibrin to form γ-glutamyl-Є-lysyl amide cross links between fibrin molecules to form an insoluble clot.

196
Q

Transfusion transmitted infections are now most likely to occur in Australasia following the infusion of which one of the following:

A. red blood cells.

B. platelets.

C. factor VIII concentrate.

D. fresh frozen plasma.

E. anti D immunoglobulin.

A

B. Bacterial contamination of blood products, especially in platelets that are stored at room temperature, is the most common infectious risk of blood transfusion, occurring in approximately 1 of 2000-3000 platelet transfusions. Transfusion–transmitted sepsis, while less common, can cause severe illness and death.

197
Q

Why don’t pregnant patients with Von Willebrands disease need treating?

A

Pregnant patients with vWD usually do not require treatment because of the natural physiologic increase in vWF levels

198
Q

Which drug is useful in the treatment of mild von Willebrands disease?

A

DDAVP is useful in the treatment of mild bleeding in most cases of type 1

199
Q

What are the lab findings in Von Willebrands disease?

A

vWF activity (by ristocetin co-factor assay) and Ag are mildly depressed, whereas the vWF multimer pattern is normal.

Except in the more severe forms of vWD that feature a significantly decreased factor VIII activity, the aPTT and PT in vWD are usually normal.

200
Q

What are the clinical findings of vonWillebrand disease?

A

Type 1 vWD usually have mild or moderate platelet-type bleeding (especially involving the integument and mucous membranes).

201
Q

What is the main type of vonWillebrands disease? What proportion does it make up?

A

Between 75% and 80% of patients with vWD have type 1. It is a quantitative abnormality of the vWF molecule that usually does not feature an identifiable causal mutation in the vWF gene.

202
Q

What is von Willebrand factor? What does it do?

A

vWF is an unusually large multimeric glycoprotein that binds to its receptor, platelet glycoprotein Ib, bridging platelets together and tethering them to the subendothe- lial matrix at the site of vascular injury. vWF also has a binding site for factor VIII, prolonging its half-life in the circulation.

203
Q

What is the most common inherited bleeding disorder?

What does the factor do normally?

A

The most common inherited bleeding disorder.

von Willebrand factor aggregates platelets and prolongs the half-life of factor VIII.

204
Q

What is the significance of the lupus anticoagulant other than it’s presence in connective tissue disease?

Which test confirms it’s presence?

A

May occur with increased frequency in individuals with underlying infection, inflammation, or malignancy, and they also can occur in asymptomatic individuals in the general population. A prolongation in the aPTT is observed that does not correct completely on mixing.

Russell Viper Venom test.

205
Q

How does uremia cause bleeding?

A

Damages platelets via –> Increased Nitric oxide; decreased release of granules

206
Q

How do salicylates cause bleeding?

A

Irreversible inhibition of platelet cyclooxygenase

207
Q

How do NSAIDS cause bleeding?

A

Reversible inhibition of platelet cyclooxygenase

208
Q

How do SSRIs induce bleeding?

A

Decreases serotonin in dense granules in platelets

209
Q

What is a Heinz body?

In what condition would you see them?

A

Heinz bodies appear as small round inclusions within the red cell body and are formed by damage to the hemoglobin component molecules, usually through oxidant damage, or from an inherited mutation (i.e. change of an internal amino acid residue). In the spleen, macrophages attempt to remove these, leaving bite cells.

Found in

G6PD deficiency

Alpha thalassaemia

NADPH deficiency

Liver disease.

210
Q

What is HbH?

A

The excess β chains form unstable tetramers (called Hemoglobin H or HbH of 4 beta chains) which have abnormal oxygen dissociation curves. The excess γ chains form tetramers which are poor carriers of O2 since their affinity for O2 is too high so it is not dissociated in the periphery.

211
Q

What are Howell-Jolly bodies?

A

Howell–Jolly bodies are histopathological findings of basophilic nuclear remnants (clusters of DNA) in circulating erythrocytes. During maturation in the bone marrow late erythroblasts normally expel their nuclei, but in some cases a small portion of DNA remains. Its presence usually signifies a damaged or absent spleen.

212
Q

What is autoagglutination?

A

Autoagglutination represents clumping of an individual’s red blood cells (RBCs or erythrocytes) by his or her own serum due to the RBCs being coated on their surface by antibodies. Happens in COLD haemagglutinin disease.

213
Q

Which haematological malignancy is AIHA most commonly associated with?

A

CLL

214
Q

What is a schistocyte?

What causes it?

A

Fragmented red blood cell

Also known as ‘helmet cell’

Associated with microangiopathy

Most commonly mechanical valves or AS

HUS

TTP

DIC

215
Q

What is the general pathogenesis of thrombotic microangiopathy?

TTP?

HUS?

A

Thrombotic microangiopathy

Pathogenesis

Platelet aggregation due to either endothelial damage (HUS) or protease insufficiency (ADAMST-13, TTP) —> shearing of circulating erythrocytes —> thrombocytopaenia and haemolytic anaemia.

TTP —> Autoantibodies against ADAMST-13, a protease that cleaves ultra-large vWF multimers —> accumulation of uncleaved multimers —> TTP

(Congenital TTP is a gene mutation resulting in decreased levels of Adamst-13)

Adults

HUS —> damage to endothelial cells (E.coli, cancer, stem cell transplant, HIV, drugs) —> platelet aggregation.

Kids

216
Q

What are the clinical findings in TTP and HUS? What is required for diagnosis?

A

For diagnosis:

Microangiopathic haemolytic anaemia

Thrombocytopaenia

Pentad (not req for dx)

Microangiopathyc haemolytic anaemia

Thrombocytopaenia

Fever

Renal insufficiency

Neurological abnormalities

Recent diarrhoeal illness (HUS)

217
Q

What are the lab findings in TTP and HUS?

A

Haemolysis: elevated LDH, bili, low haptoglobin, reitculocytosis, negative DAT, schistocytes on blood smear)

Thrombocytopaenia

ARF (usually HUS)

Positive stool culture for E.coli or shiga-toxin

Coagulation studies are normal

218
Q

What is the treatment for TTP and HUS?

A

Plasma exchange and replace w/ FFP —> essential, 95% mortality without it

Supportive only if diarrhoea-HUS

Continue to exchange for two days until platelet count and LDH are normal for at least 2 days.

Give FFP if no plasma available

Platelet transfusions are CONTRAINDICATED as may make it worse

Dialyse if significant renal failure

Relapse —> Plasma exchange

219
Q

What the main differences between TTP and HUS?

A

TTP —> autoantibody to ADAMST-13

Usually has fever and neurological defects

HUS —> Endothelial damage - usually due to e.coli

Usually has renal failure, no fever or neurological defects

Recent history of diarrhoea

220
Q

What is the pathogenesis of hereditary spherocytosis?

A

Familial defect of red cell membrane due to an abnormality in spectrin, actin or other proteins.

Scaffolding defect results in decrease surface to volume ratio and loss of central pallor of the RBC —> less deformability —> trapping of RBCs in the spleen —> removal by splenic macrophages.

221
Q

What are the clinical findings in hereditary spherocytosis?

A

Autosomal dominant

Variable anaemia

May have aplastic crisis due to infection (Parvo B19) or folate deficiency

Gallstones from a young age

Jaundice

Icterus

Palpable spleen

222
Q

What are the lab finding in hereditary spherocytosis?

A

Smear: Reticulocytosis always, large amount of spherocytes on the blood film, microcytosis

Increased MCHC

Negative coombs

Increased osmotic fragility (abnormal vulnerability to swelling induced by hypotonic media)

223
Q

What is the treatment of hereditary spherocytosis?

A

Daily folic acid

Splenectomy (if severe) cures it (stops destruction, doesn’t cure abnormal RBCs)

224
Q

What is the pathogenesis of paroxysmal nocturnal haemoglobinuria?

A

Pathogenesis

Acquired defect in PIG-A (phosphatidylinositol class A) gene —> deficient GPI anchor for cell membrane protein —> deficient complement regulating proteins CD55 and CD59 —> unregulated formation of MAC attack complex on RBC membranes —> intravascular haemolysis.

Free haemoglobin is released into the blood —> scavenges nitric oxide —> oesophageal spasm, erectile dysfunction, renal damage, thrombosis (primary cause of death)

In short - abnormal sensitivity of the RBC membrane to complement.

225
Q

What are the clinical findings in PNH?

A

Episodic haemoglobinuria on first morning urine (pH drops at night —> haemolysis)

Thrombosis (mesenteric and hepatic veins, CNS veins (sagittal), skin vessels (painful nodules)

May arise on it’s own or in setting of aplastic anaemia with progression to myelodysplasia or AML

226
Q

What are the lab findings in PNH?

A

Variable anaemia

May or may not have reticulocytosis

Smear is non diagnostic - may have macro-ovalocytes and polychromasia

Urine haemosiderin (chronic haemolysis)

Iron deficiency due to haemoglobinuria

Flow cytometry: Deficiency of CD55 and 59

Low WBC and PLT, esp if aplastic anaemia

227
Q

What is the treatment of PNH?

A

Mild - do nothing

Severe or thrombosis —> Eculizumab

Aplastic anaemia or MDS —> allo STC

Replace iron

Corticosteroids can be useful

228
Q

What is eculizumab and what are the benefits?

A

Anti-terminal complement protein C5 (initiator of MAC) - used for PNH.

Improves QOL, reduces haemolysis, transfusion requirements and thrombosis risk

229
Q

What is the pathogenesis of G6PD deficiency?

A

Hereditary enzyme (G6PD) defect (deficiency) —> excess oxidised glutathione —> haemoglobin denatures —> precipitates Heinz bodies causing RBC membrane damage —> premature removal of RBCs by spleen (extravascular haemolysis)

230
Q

What are the clinical findings in G6PD deficiency?

A

X-linked

Found in Africans (milder), and Mediterraneans (severe), and Ashkenazi Jews.

Episodic haemolysis in response to oxidative stress on RBCs (infection, exposure to certain drugs

231
Q

Why is the haemolysis in G6PD deficiency often self limiting?

A

Self limited haemolysis because old RBCs w/ low enzyme activity are removed and replaced with young RBCs (reticulocytes) which are G6PD replete.

Severe deficiency (Mediterranean variant) —> chronic haemolytic anaemia.

232
Q

What are the lab findings in G6PD deficiency?

A

Between episodes —> normal blood

During —> Hb rarely drops below 80, reticulocytosis, increased bill

Smear: Bite/blister cells (pitting of Hb aggregates by the spleen). Heinz bodies on cresyl violet stain

Low G6PD levels BETWEEN episodes (false normal during or shortly after due to replete retics), check weeks later.

Severe disease - permanently low enzyme levels

233
Q

What is the treatment of G6PD defiency?

A

Avoid oxidant drugs

234
Q

What drugs should you avoid in G6PD deficiency and why?

A

They are oxidant drugs causing oxidant stress –> haemolysis.

Drugs to avoid: Dapsone (leprosy), methylene blue, phenazopyridine, primaquine (malaria, PCP), rasburicase (gout, tumor lysis syndrome), toluidine blue, nitrofurantoin

235
Q

What is the pathogenesis of sickle cell anaemia?

A

Autosomal recessive disorder —> DNA base change of valine for glutamine on the B-globin change —> Bs —> A2BS2 —> HbS —> unstable, polymerises under stress (dehydration, acidosis, hypoxia) —> sickled RBCs —> splenic sequestration —> haemolytic crises & eventual splenic infarction

* HbF cannot polymerise —> retards sickling when present

236
Q

What are the clinical findings in sickle cell anaemia?

A

Onset in 1st year of life (HbF levels fall due to y-globin switching to B-globin

Chronic haemolytic anaemia —> jaundice, pigment gallstones (young) splenomegaly (young), poorly healing ulcers over lower tibia

Life threatening severe anaemia during haemolytic or aplastic crises (viral or other infections, folic acid deficiency)

Acute painful episodes due to acute vaso-occlusion

Delayed puberty, increased incidence of infection due to hyposlenism

Chronically ill, jaundice, hepatomegaly but no splenomegaly

Cardiac enlargement with hyper dynamic and systolic murmurs

Nonhealing lower leg ulcers

237
Q

What are the features of the vaso-occlusive disease in sickle-cell anaemia?

A

Acute painful episodes due to acute vaso-occlusion from clusters of sickled cells (spontaneous, infection, dehydration, hyopxia)

Pain —> back and long bones, chest, lasts hours to days

Acute vaso occlusion —> sagital sinus thrombosis (strokes), priapism, organ and bone necrosis, renal medullary papillae infarcts —> gross haemoturia (more in trait), retinopathy, plum HT Not assoc w/ increased haemolysis

Ischaemic necrosis of bone due to vascular occlusion —> salmonella osteomyeletis

238
Q

What are the lab findings of sickle cell anaemia?

A

Lab findings

Chronic haemolytic anaemia

Smear: Irreversibly sickled cells, nucleated RBCs, hyposplenism: Howell-Jolly bodies and target cells

Elevated WBC and reactive thrombocytosis

Elevated indirect bili

Hb EPG (diagnostic): HbS comprises 85-98% of the haeoglbonin

Homozygous - no HbA present, elevated HbF (high = better prognosis)

239
Q

What is the treatment of sickle-cell anaemia?

A

Allo SCT is curative >80% of kids

Hydroxyurea increases HbF levels epigenetically, reduces painful crises and overall survival

Supportive care is mainstay - folic acid, transfuse in aplastic or haemolytic crises, pneumovax.

Hydrate and oxygenate through acute painful episodes.

Acute vaso-occlusive crises (pain, acute chest, priapism, stroke) —> exchange transfusion

240
Q

What is the clinical significant of sickle-cell trait?

A

Heterozygotes —> may be positive on screening test, HbS 40% on EPG

Haematologically normal

May experience sudden cardiac death and rhabdo during vigorous exercise esp at high altitude

Increased risk of VTE

May have renal medullary disease as it’s acidotic —> microscopic and gross haematuria.

No treatment.

241
Q

What is the mechanism of autoimmune haemolytic anaemia?

A

Acquired IgG autoantibody binds RBC at body temp —> Antibody directed at basic component of Rh system present on most human RBCs —> splenic macrophage recognises Fc portion of antibody —> removal of RBC membrane by macrophage —> spherocyte formed —> decreased deformability -> trapped in splenic red pulp —> haemolysis

242
Q

What is the second mechanism of haemolysis in autoimmune haemolytic anaemia?

A

If large amounts of IgG —> fixation of complement —> presence of C3b on RBC —> destruction of RBC by hepatic Kupffer cells

(Direct complement driven lysis anywhere else is rare).

243
Q

What is the aetiology of autoimmune haemolytic anaemia?

A

50% idiopathic

Rest: SLE, CLL, or lymphoma

Drug induced: Antibody is against membrane-drug complex. (Penicillim ceftriaxone, piperacllin, cefotetan, and fludarabine (immunosuppresson —>defective immune surveillance)

244
Q

What are the clinical findings in autoimmune haemolytic anaemia?

A

Rapid onset haemolytic anaemia

Fatigue and dyspnoea

Angine or cognitive heart failure

Jaundice and splenomegaly

245
Q

What are the lab findings in autoimmune haemolytic anaemia?

A

Variable anaemia, may be v severe

Reticulocytosis and spherocytosis on smear, nucleated RBCs if severe

Indirect hyperbilirubinaemia

10% have concurrent immune thrombocytopaenia (Evans syndrome)

DAT positive for IgG +/- C3b

246
Q

What is Evans syndrome?

A

AIHA + concurrent immune thrombocytopaenia

247
Q

What is the difference between the direct antiglobulin test (Coombs) and indirect?

A

DAT: rabbit IgM antibody against Human iG or complement —> agglutination = presence of antibody on RBC surface

Indirect: patient SERUM mixed with type O RBCs —> agglutination = presence of free antibody

Positive IDAT = large amount of autoantibody that has saturated RBC binding sites, winding up in the serum.

248
Q

What is the treatment of AIHA?

A

Autoantibody makes it difficult to get a compatible cross-match

Pred 1mg-2mg/kg

Transfused RBCs will survive similarly to the patient’s own

Splenectomy potentially curable

Rapid + severe —> therapeutic plasmapheresis

Rituximab may be effective

249
Q

What is the prognosis for AIHA like?

A

The long-term prognosis for patients with this disorder is good, especially if there is no underlying autoimmune disorder or lymphoma.

250
Q

What is the pathogenesis of cold agglutinin disease?

A

IgM autoantibody (cold agglutinin) against I antigen on RBCs —> binds and fixes complement at peripheries (cool areas) —> RBC returns to warmer circulation —> IgM dissociates —> only complement (C3b) left behind —> recognised by hepatic Kupffer cells —> RBC sequestration and destruction, sometimes MAC attack

251
Q

What is the aetiology of cold agglutinin disease?

A

Idiopathic

Waldenstrom or CLL due monoclonal iGm paraprotein

Infectious: Mycoplasma pneumonia, mono, measles, mumps

252
Q

What are the clinical findings of cold agglutinin disease?

A

Mottled or numb fingers or toes

Episodic low back pain

Dark coloured urine one exposure to cold

Haemolytic anaemia is rarely severe

253
Q

What are the lab findings in cold agglutinin disease?

A

Mild anaemia w/ reticulocytes

Agglutinated RBCs on smear at room temp

DAT + for C3b only

Serum cold agglutinin titer semi-quantitative

254
Q

What is the treatment of cold agglutinin disease?

A

Avoid cold!

Splenectomy and pred ineffective unless underlying lymphoproliferative disease (haemolysis takes place in liver and blood)

Rituximab is treatment of choice, relapse is effectively retreated

May need to use alkylators in severe disease

May be difficult to find compatible blood

TRANSFUSE THROUGH A BLOOD WARMER

255
Q

What is the hallmark of microangiopathic haemolytic anaemia?

A

The finding of fragmented red blood cells (schistocytes, helmet cells) on the peripheral blood smear.

The anemia is intravascular, producing hemoglobinemia, hemoglobinuria and, in severe cases, methemalbu-minemia

256
Q

Is the anaemia of cold-agglutinin disease intravascular or extravascular?

A

Mainly extravascular –> liver

Intravascular due to fixation of complement

257
Q

What is the mechanism of iron deficiency in chronic microangiopathic haemolytic anaemia?

A

Continuous low-grade haemoglobinuria

258
Q

Question 66

A 45-year-old woman presents with progressive lethargy over 10 days. There is no history suggestive of infection. There has been no previous medication.

Initial investigations show:

Which one of the following is most likely to be present on the blood film?

A. Plasma cells.

B. Red cell autoagglutination.

C. Red cell fragmentation.

D. Hypersegmented neutrophils.

E. Target cells.

A

Answer: B - cold agglutinin disease is DAT positive of complement only

259
Q

Question 69

A 38-year-old woman presents with a one-week history of dyspnoea on exertion. She has had no previous medical problems and is taking no medications. Physical examination is unremarkable apart from mild pallor and a palpable splenic tip. Initial investigations are as follows:

The blood film is shown below.

Which one of the following conditions is the most likely cause of these findings?

A. Hereditary spherocytosis.

B. Systemic lupus erythematosus.

C. Infectious mononucleosis.

D. Mycoplasma infection.

E. Cold haemagglutinin disease.

A

Answer: B

This is autoimmune haemolytic anemia - there are minimal spherocytes on the film so it’s not A - and C, D, and E all cause cold agglutinin disease and are only positive for C3 on the DAT as the Ig hs dissociated (it’s IgM anyway)

260
Q

A 69-year-old woman commences a chronic transfusion program for anaemia secondary to myelodysplasia. She experiences a non-haemolytic febrile transfusion reaction during each of two red cell transfusions administered four weeks apart.

Which one of the following reasons best explains the benefit of using filtered blood to prevent further reaction?

Reduced exposure to:

A. donor leucocytes.

B. donor cytokines.

C. bacterial antigen.

D. cytomegalovirus (CMV).

E. donor isohaemagglutinins.

A

A. donor leucocytes.

261
Q

A 62-year-old woman underwent an elective cholecystectomy. Routine full blood examination revealed:

Physical examination revealed no lymphadenopathy and no hepatosplenomegaly. The woman was asymptomatic. Flow cytometry revealed that the lymphocytosis marked as CD5+, CD19+ and CD23+.

Which one of the following is the most appropriate management option?

A. Observation alone.

B. Prednis(ol)one therapy.

C. Chlorambucil therapy.

D. Combination chemotherapy.

E. Autologous bone marrow transplantation.

A

Answer - A, this is asymptomatic CLL.

262
Q

A 65-year-old man with a history of osteoarthritis and mild chronic obstructive pulmonary disease presents with several days of nausea and vomiting and is now confused, febrile and icteric. He has extensive bruising and some oral mucosal bleeding. His medications are intermittent piroxicam and a salbutamol inhaler.

His full blood count shows:

Blood film as shown in the photograph below.

The most likely diagnosis is:

A. gram-negative sepsis.

B. leptospirosis.

C. disseminated intravascular coagulation.

D. thrombotic thrombocytopenic purpura.

E. piroxicam toxicity.

A

Answer: D

This is TTP - there is fragmented red cells (hallmark) and thrombocytopaenia due to platelets clumping. The absence of a recent diarrhoeal infection and presence of neurological symptoms makes it more likely to be TTP over HUS (if that were an option).

263
Q

What is the hematocrit?

A

Percentage of red blood cells in the full blood count.

It is usually 45% for men and 40% for women.

264
Q

What is anisocytosis?

A

Variations in red blood cell size

265
Q

What is poikilocytosis?

A

Variations in red blood cell shape

266
Q

What is the difference between horse and rabbit ATG?

A

Horse is twice as effective, rabbit is not used in Australia.

267
Q

Which drug causes a delayed neutropaenia of around 2-3 months?

A

Rituximab

268
Q

What are the central causes of pancytopaenia?

A

Drugs

B12/folate deficiency

• AA

MDS

Haem malignancy

Myelofibrosis

Non-haem infiltration

– Solid tumour

– Storage disorders

Haemophagocytic Syndrome

269
Q

What are the peripheral causes of pancytopaenia?

A

Peripheral Destruction

  • Hypersplenism
  • Dilutional (pregnancy)
  • Severe sepsis
270
Q

Which is the only cause of pancytopaenia that does not result in a hypercellular bone marrow?

A

Aplastic anaemia (any cause)

271
Q

What is the epidemiology of aplastic anaemia?

A
  • RARE – 1-2/million)
  • 3 peaks

– 2-5 yrs (inherited)

– 20-25 yrs

– >55 yrs (acquired)

272
Q

What is the clinical presentation of aplastic anaemia?

A

– Bleeding

– Fatigue & pallor

– Fevers/mouth ulcers/bacterial infections

273
Q

What is the basic pathogenesis of aplastic anaemia?

A

Stem cell failure – inherited or acquired

274
Q

What are the causes of childhood aplastic anaemia?

A
  • 75% inherited
  • Fanconi Anaemia (FA)
  • Dyskeratosis Congenita (DC)
  • Congenital Amegakaryocytic

Thrombocytopenia (CAMT)

• Schwachmann-Diamond Syndrome (will

be discussed under neutropenia)

275
Q

What are the pathogenesis and features of Fanconi’s anaemia?

A

AR or X linked

All involved in DNA repair & stabilisation - 8 of these form complex in response to replicative stress, esp crosslinking and reactive O2 species

All ethnic/racial groups (more in Ashkenazi)

• congenital malformations in 60-70%

– Café-au-lait spots & hypopigmentation

– Abnormal thumbs

– Microcephaly & short stature

– Hypogonadism & delayed development

  • Bone marrow failure, MDS –> AML (later)
276
Q

What is the natural history of fanconi’s anaemia?

A

By age 40 progression to:

  • Complete BM failure 90%
  • Acute Leukaemia/MDS 33%
  • Solid CA 28%

– SCC head & neck & vulva commonest

• 80% mortality by age 40yrs

277
Q

What is the treatment of Fanconi’s anaemia

A

Fanconi Anaemia - Treatment

• Allogeneic Stem Cell Transplant – curative

– problem of affected family donors (harder to

find unaffected but HLA-matched donor)

– May inc risk solid tumours

• Androgens (oxymethalone)

– If no transplant available

– 50% respond

– Anaemia responds best

• Supportive care – transfusion, GCSF

278
Q

What are the features of dyskeratosis congenita?

A

Classic triad of ectodermal abnormalities

  • reticulated rash face/shoulders/trunk
  • mucosal leukoplakia
  • progressive nail dystrophy, small thin bed, longitudinal ridges, nails disappear with age

75% have one, 46% have all 3 at Diagnosis

Childhood cause of aplastic anaemia

279
Q

How do drugs cause pancytopaenia?

A

Drugs as cause of pancytopenia

• Direct toxic effect of drugs

– eg chemotherapy or immunosuppression

– commonest cause of pancytopenia

– Recovery in days-weeks

– G-CSF speeds recovery when some

precursors present

• Adverse interaction

– Eg azathioprine + allopurinol

– Eg Methotrexate in renal impairment

280
Q

What are the drugs that cause aplastic anaemia?

A
  • Idiosyncratic effect of drugs
  • Rare effect
  • Cause unclear in most cases

– Chloramphenicol 1:20,000

– Gold (dose-related >200mg)

– NSAIDs

– Sulphonamides

– Antiepileptics

– Arsenics

281
Q

What are the non-drug causes of acquired aplastic anaemia?

A

– Irradiation

– Viral – Parvo B19 (more commonly causes PRCA) ,

Hepatitis, HIV, EBV

– Chemical – benzene (rubber factories), lindane, glue

vapours

– Immune – eosinophilic fasciitis, SLE, GvHD

PNH – 1/3 will develop AA

– Pregnancy – usually resolves with delivery

– Thymoma/thymic CA

– But most adult cases are IDIOPATHIC (80%)

282
Q

What is the basis for aplastic anaemia having an underlying immune cause?

A

– Most respond to immunotherapy

– Assoc w rare immune eosinophilic fasciitis

– AA can develop in GvHD

283
Q

What are the outcomes in aplastic anaemia for allogeneic BMT vs Immunosuppression?

A

15 year surivival 69% for BMT vs 38% for immunosuppression

Much less risk of malignancy with BMT, higher risk of MDS/AML with immunosuppression

HOWEVER
only 30% will have matched donor, risk goes up with age, and GvHD with BMT

284
Q

What is the prognosis in aplastic anaemia?

A

With immunosuppression:

Best case around 50% in over 60s and 72% in under 50s

Worst, 21% and 50% respectively

285
Q

Who do you give an allo-BMT to in aplastic anaemia?

A

<20 years old if available, up to age 50 if no comorbidities

Avoid pre-transfusion wherever possible.

286
Q

What is the treatment for relapsde and refractory aplastic anaemia?

A

Relapse – 50-60% respond to retreat with

ATG (horse or rabbit), CAMPATH similar

• Refractory – 30% respond to rabbit ATG,

CAMPATH similar

• OS in responders same as those with

initial response but no relapse

287
Q

What is campath/alemtuzumab?

A

Monoclonal antibody that binds to CD52, (mature lymphocytes), but not stem cells.

288
Q

What is the septic risk in isolated neutropaenia?

A

Blood neuts = 3% total body neutrophil

reserve => markers of reserve can

indicate septic risk

  • Febrile, raised CRP, gingivitis, mucositis – poor reserve – high septic risk
  • Abscess formation – good reserve – lower septic risk
289
Q

When is septic risk greatest in neutropaenia?

A

Septic Risk relatively low down to

around 0.5 x 10^9/L, then rises sharply

• Risk is greater in context of:

– a falling count (progressively decreasing availability)

– lack of early precursors/empty BM rather than when maturation arrest)

– prolonged length of neutropenia

290
Q

What is the most common fatal cause of transient neutropaenia?

A

Drug idiosyncratic

V rare 1-10/million

May be v severe

Direct toxic effect on BM orimmune destruction

291
Q

What are the commonly implicated drugs in drug induced neutropaenia?

A

Carbimazole

NSAIDS

Sulphonamides

Antiepileptics

Captopril

Dapsone

PTU

Rituximab

292
Q

When should you give G-CSF in neutropaenia?

A

When precursors appear in the bone marrow, without them, G-CSF will not work

293
Q

What is the mechanism of dialysis induced neutropaenia?

A

Haemodialysis –

– Severe neutropenia on blood taken during dialysis

– Complement activation on dialysis membrane causes attachment of neutrophils onto lung endothelium

– Not true neutropenia - repeating the test off dialysis shows a normal neutrophil count

294
Q

Aside from drugs, what are the two very clinically relevant causes of transient acquired neutropaenia?

A

Bacterial Sepsis –

  • destruction exceeds production,
  • bad prognostic indicator
  • Commonest in neonates

Neonatal Isoimmune –

– transplacental passage of maternal Ab to paternal

neutrophil Ag (cf Rh HDN).

– Can be severe and prolonged (ie months after birth)

– May respond to G-CSF

295
Q

What are the features of chronic benign neutropaenia?

A

Chronic Idiopathic Neutropenia/ Benign

Chronic Neutropenia

– Neutrophil equivalent to adult ITP

– May be seen with SLE or other AI diseases

Most are mild & never require Rx

(Can do antibody testing but no one does)

296
Q

What is ‘Large Granular Lymphocyte’ Syndrome?

A

– Neutropenia associated with increased circulating LGLs (NK cells)

– Total lymphocyte count often normal

– Not malignant

– May regress spontaneously

One of the causes of acquired neutropaenia - it is a chronic condition

297
Q

What is Felty’s Syndrome?

A

– Commonest mild-mod chronic neutropenia

– Triad of seropositive RA, neutropenia and splenomegaly (but only 25% have

splenomegaly)

– about 1% RA patients

– Probably AI basis (higher RF & ANA)

– Can occur in ‘burnt-out’ RA

– Often have “LGL’ syndrome

298
Q

What is Severe Congenital Neutropaenia?

A

– Life threatening neutropenia from birth

– Neutrophils < 0.5

– RARE - 2/million

AR/AD -mixed bag of molecular defects found eg ELANE, HAX-1, 40% not known

HAX-1 maintains mitochondrial membrane & prevents myeloid apoptosis

@15yrs G-CSF– 10% death by sepsis, 22% AML

HSCT curative

299
Q

What is Chediak-Higashi Syndrome?

A

Oculocutaneous albinism,

Severe neutropenia & life-threatening

pyogenic infections (most die by 7yr)

Mild coagulopathy

Progressive neurological deterioration –

pp neuropathy (if don’t die of infection)

Accelerated phase – lymphohistiocytosis

usually fatal

300
Q

What is WHIM (lazy leucocyte syndrome?)

A

Warts, Hypogammaglobulins, Infections, Myelokathexis)

– CXCR4 mutation

– Abnormal apoptosis & migration (hypercellular BM)

– Rarely fatal

• Myelokathexis can also occur alone

301
Q

What is cyclic neutropaenia?

A

– Neuts cycle over 3 weeks (14-35d) –consistent within individual, lessens with age

– 1-2/million

– AD - Neutrophil elastase (ELANE) mutations (Chrom 19p13.3)

– Usually mild but can be severe (death)

– regular G-CSF reduces nadir and infections

– NO somatic abnormalities, NO inc risk malignancy

302
Q

What are the acquired transient causes of neutropaenia?

A

– Drug

– Viral

– Bacterial

– nutritional

– Iso-immune

303
Q

What are the acquired chronic causes of neutropaenia?

A

– Benign chronic neutropenia

– LGL syndrome

– Felty’s syndrome

304
Q

What are the inherited causes of neutropaenia?

A

– Severe congenital neutropenia

– Cyclical neutropenia

305
Q

What is the primary concern in febrile neutropaenia?

A

Enteric gram negative organisms

306
Q

What is the primary concern in prolonged neutropaenia?

How should you treat it?

A

Fungal infections - candida or aspergillus

Treat with azoles or echinocandins (the ‘fungins’ ie caspofungin)

307
Q

What are the clinical features of significant neutropaenia?

A

Stomatitis and in infections due to gram-positive or gram-negative aerobic bacteria or to fungi such as Candida or Aspergillus.

308
Q

What is the treatment for aplastic anaemia?

A

Mild —> EPO, transfusions and antibiotics as necessary

Severe —:> ALLO BMT is curative

Over 40/ or no HLA match —> horse ETG plus cyclosporine or tac

MUST give with pred to avoid serum sickness and infusion reactions

Need to give ongoing cyclosporine for several years post ATG

Androgens can be considered as may have some response.

309
Q

What are the two main proposed mechanisms behind aplastic anaemia?

A

Immunological

Telomere length maintenance defects

310
Q

Which groups have lower limit of normal neutrophil count ranges but are healthy?

Higher?

A

Lower –> blacks and asians

Higher –> third tri pregnant women

311
Q

What is the pathophysiology of aplastic anaemia?

A

Haematopoetic stem cell injury (congenital: telomere length maintenance defect, acquired: autoimmune) —> marrow hypoplasure —> failure to produce all mature blood cells —> pancytopaenia

312
Q

What are the factors that predict response to immunological treatment of aplastic anaemia ?

A

Age,

Reticulocyte count,

Lymphocyte count,

Age-adjusted telomere length of leukocytes at the time of diagnosis.

313
Q

What is hemaphagocytic syndrome?

A

Severe hyperinflammation caused by uncontrolled proliferation of activated lymphocytes and macrophages characterised by proliferation of morphologically benign lymphocytes and macrophages that secrete high amounts of inflammatory cytokines. It is classified as one of the cytokine storm syndromes.

314
Q

What are the clinical manifestations of haemophagocytic syndromes?

A

HLH clinically manifests with fever, hepatosplenomegaly, lymphadenopathy, jaundice and a rash.

315
Q

What are the lab findings of haemophagocytic syndrome?

A

The blood count typically shows pancytopenia — anemia, neutropenia and thrombocytopenia.

The blood film may show hemophagocytosis.

The liver function tests are usually elevated. Hypoalbuminemia is common.

The serum C reactive protein, erythrocyte sedimentation rate and ferritin level are markedly elevated.

The serum fibrinogen level is usually low and the D-dimer level is elevated.

The sphingomyelinase is elevated.[4]

BMB - histiocytosis

316
Q

What are the causes of secondary haemophagocytic syndrome?

A

Secondary haemophagocytic lymphohistiocytosis (acquired haemophagocytic lymphohistiocytosis) occurs after strong immunologic activation, such as that which can occur with systemic infection, immunodeficiency, or underlying malignancy. Both forms are characterized by the overwhelming activation of normal T lymphocytes and macrophages, invariably leading to clinical and haematologic alterations and death in the absence of treatment.

317
Q

What is the cause of primary haemophagocytic syndrome?

A

Autosomal recessive with 5 genetic subtypes.

318
Q

What are the most common infections associated with neutropaenia?

A

Most common infections: Sepsis, cellulitis, pneumonia, and fever

319
Q

Other than cytotoxic chemotherapy, which drugs can cause neutropaenia?

A

Sulfonamides, chlorpromazine, procainamide, penicillin,

cephalosporins, cimetidine, methimazole, phenytoin, chlorprop-

amide, antiretroviral medications, rituximab

320
Q

What’s in prothrombinex and when is it used?

A

Provision of factors II, IX and X

  • bleeding and warfarinized
  • bleeding from factor deficiency
  • prophylaxis
321
Q

Question 52

A 58-year-old previously well man presents with a two-week history of spontaneous bruising and a large, painful left thigh haematoma following minimal trauma. He is on no medication and his blood count has been documented as normal 18 months earlier prior to elective surgery. There is no previous history of bleeding with trauma or with surgery and no family history of bleeding disorder. Physical examination is normal apart from scattered ecchymoses and the swollen, bruised left thigh. The haematoma is confirmed on ultrasound examination.

Investigations show:

The most likely diagnosis is:

A. haemophilia A.

B. haemophilia B.

C. primary antiphospholipid antibody syndrome.

D. von Willebrand’s disease.

E. acquired factor VIIIc inhibitor.

A

Answer: E

322
Q

Question 96

A 62-year-old previously well woman is referred for evaluation of recurrent mouth ulceration and several bacterial skin and urinary tract infections over 12 months, with one episode requiring parenteral antibiotic therapy. She is on no medication apart from intermittent antibiotics. Physical examination shows some mouth ulcers and pustular skin lesions. Full blood count (as follows) is essentially unchanged compared with previous counts measured three times during the preceding few weeks.

Investigations show:

Antinuclear and rheumatoid factors are negative and a marrow examination is unremarkable except for left- shifted granulopoiesis.

The most appropriate therapy is:

A. granulocyte colony stimulating factor (G-CSF).

B. lithium.

C. intravenous gamma globulin.

D. granulocyte transfusions from a family member.

E. long-term prophylactic antibiotics.

A

A

323
Q

QUESTION 7

A 64-year-old man is taking oral prednis(ol)one (7.5 mg/day) for polymyalgia rheumatica. He reports progressive lethargy, shoulder girdle pain and stiffness and also recent indigestion.

Full blood examination reveals:

Blood film shows normocytic and microcytic red cells with mild hypochromasia and rouleaux, as well as neutrophilia, lymphopenia, monocytosis and thrombocytosis

Iron studies reveal:

What is the most likely cause of the patient’s anaemia?

A. Chronic inflammation.

B. Beta thalassaemia trait.

C. Iron deficiency.

D. Autoimmune haemolysis.

E. Sideroblastosis.

A

Answer A

324
Q

QUESTION 28

A 26-year-old previously well woman presents with several weeks lethargy and the recent onset of pallor and dark urine. She is on no medication except an oral contraceptive. On examination she is afebrile, anaemic, slightly icteric and has mild splenomegaly.

Full blood examination reveals:

Her blood film is shown below.

The most appropriate next investigation is:

A. Heinz bodies preparation.

B. glucose-6-phosphate dehydrogenase level.

C. unconjugated bilirubin.

D. direct antiglobulin (Coombs’) test.

E. urinary haemosiderin.

A

Answer: D

325
Q

QUESTION 57

A 48-year-old previously well man is receiving a transfusion of four units of packed red cell concentrate via a peripheral intravenous line after presenting with melaena secondary to a bleeding duodenal ulcer. He has not been previously transfused. Admission biochemistry (including urea, electrolytes and creatinine) is normal and pre-transfusion blood counts are normal apart from anaemia (haemoglobin 64 g/L [135-170]). Less than five minutes after transfusion of the third unit of blood is commenced, he complains of feeling very unwell and abruptly develops fever, chills, rigors and profound hypotension.

In the absence of ABO, Rhesus or minor blood group donor-recipient incompatibility, the most likely diagnosis is:

A. bacterial contamination of the transfused blood.

B. transfusion-associated graft-versus-host disease.

C. electrolyte disturbance.

D. citrate toxicity.

E. reaction to plasticiser in infusion bag.

A

A

326
Q

QUESTION 8

A 65-year-old woman presents for inguinal hernia repair. She is otherwise well, apart from several years of hypertension, currently controlled by captopril. There is no history of excessive infection. Physical examination reveals borderline cervical lymphadenopathy.

A preoperative full blood count shows:

The blood film is shown in the photograph below.

The most appropriate initial investigation for diagnosis is:

A. a computed tomography (CT) scan of the neck, chest, abdomen and pelvis.

B. Epstein-Barr virus serology.

C. peripheral blood cytogenetic analysis.

D. bone marrow examination.

E. immunophenotypic analysis of peripheral blood mononuclear cells.

A

E

327
Q

QUESTION 33

A 70-year-old woman with chronic lymphocytic leukaemia and mild chronic airways disease presents for routine review. In the past eight months, she has been hospitalised twice for community-acquired pneumonia.

Examination reveals minor cervical lymphadenopathy. There is no peripheral lymphadenopathy elsewhere and no hepatosplenomegaly.

The most appropriate treatment is:

A. prophylactic antibiotics.

B. regular granulocyte colony-stimulating factor (G-CSF).

C. oral corticosteroid.

D. oral chlorambucil.

E. regular immunoglobulin infusions.

A

E

328
Q

QUESTION 45

A 62-year-old man with a history of controlled hypertension presents with worsening lethargy over several months and more recently, weight loss, night sweats and easy bruising. Examination reveals anaemia, bruising and firm splenomegaly 12 cm below the left costal margin; no lymphadenopathy is evident.

Full blood examination reveals:

The blood film is shown below.

The most likely diagnosis is:

A. metastatic carcinoma.

B. megaloblastosis.

C. primary myelofibrosis.

D. systemic lupus erythematosus.

E. chronic myeloid leukaemia.

A

C

329
Q

QUESTION 58

A 72-year-old woman presents with confusion after a minor fall and is found to have a subdural haematoma on computed tomography (CT) scanning which requires urgent surgical drainage. She takes warfarin for atrial fibrillation and has been taking an antibiotic for urinary infection.

Full blood examination reveals:

In addition to parenteral vitamin K therapy, she should receive:

A. fresh frozen plasma.

B. cryoprecipitate.

C. recombinant factor VIIIc.

D. packed red cells.

E. platelet concentrate.

A

Answer: A

330
Q

QUESTION 84

A 63-year-old woman is recovering from induction chemotherapy commenced 28 days previously for acute myeloblastic leukaemia. She is receiving granulocyte colony-stimulating factor (G-CSF) and is no longer neutropenic. A Hickman’s catheter is in situ. She remains intermittently febrile and Candida albicans continues to be cultured from samples obtained from the Hickman’s catheter and peripheral blood, despite therapy with intravenous amphotericin B (1 mg/kg/day) from day 20.

The most appropriate course of action is to:

A. increase the daily dose of amphotericin B.

B. arrange granulocyte transfusion.

C. change to liposomal amphotericin.

D. exclude hepatosplenic candidiasis radiologically.

E. remove the Hickman’s catheter.

A

E. remove the Hickman’s catheter.

331
Q

QUESTION 20

A 36-year-old previously well woman presents with lethargy of two to three weeks duration and gum bleeding over the previous week. Examination shows her to be afebrile and anaemic with scattered bruises and petechial haemorrhages. A chest X-ray is normal.

A bone marrow aspirate shows that 90% of nucleated cells have the appearance as seen in the photograph below.

The most appropriate initial therapy is:

A. idarubicin.

B. all-trans-retinoic acid.

C. vincristine and prednis(ol)one.

D. fludarabine.

E. high-dose cyclophosphamide.

A

Answer B.

The aspirate shows a hypergranular myelocyte which is characteristic of APML

332
Q

QUESTION 70

A 25-year-old previously well male student presents with recent exertional dyspnoea, epistaxis and bruising. There is no history of medication, toxic chemical exposure or infection.

Examination reveals significant anaemia with scattered bruises and petechial haemorrhages.

Full blood examination reveals:

B12, folate and iron studies are normal. Hepatitis/human immunodeficiency virus (HIV) serology is negative.

Attempted marrow aspiration results in a ‘blood tap’. A marrow trephine biopsy section is shown below.

QUESTION 70 CONTINUED

The therapy most likely to be associated with long-term remission is:

A. granulocyte colony-stimulating factor (G-CSF) and erythropoietin.

B. stem cell allograft from a human leucocyte antigen (HLA)-matched sibling.

C. oxymetholone.

D. antithymocyte globulin.

E. high-dose cyclophosphamide.

A

Answer B

333
Q

Question 84

A 78-year-old man presents with progressive lethargy for several months and the recent onset of bruising, epistaxis, headaches and visual deterioration. Physical examination shows him to be anaemic with scattered ecchymoses; he has reduced visual acuity bilaterally and fundoscopy reveals retinal venous distension with numerous retinal haemorrhages.

Investigations show:

The most appropriate initial therapy is:

A. melphalan and prednis(ol)one.

B. platelet transfusion.

C. plasma exchange.

D. renal dialysis.

E. intravenous gamma globulin.

A

E

334
Q

QUESTION 3

The predominant site of gastrointestinal absorption of dietary iron is the:

A. proximal small intestine.

B. gastric fundus.

C. terminal ileum.

D. pyloric antrum.

E. colon.

A

A

335
Q

QUESTION 25

In a transfusion-dependent patient with idiopathic acquired sideroblastic anaemia, the most appropriate treatment for prevention of transfusional iron overload is:

A. vitamin C.

B. phlebotomy.

C. desferrioxamine.

D. ethylene-diamine tetra-acetic acid (EDTA).

E. pyridoxine.

A

Answer: C

336
Q

QUESTION 35

Infusion of cryoprecipitate is principally indicated for the replacement of:

A. factor IX.

B. fibrinogen.

C. immunoglobulin A (IgA).

D. anti-thrombin.

E. protein C.

A

Answer: B

337
Q

QUESTION 58

A 72-year-old woman presents with confusion after a minor fall and is found to have a subdural haematoma on computed tomography (CT) scanning which requires urgent surgical drainage. She takes warfarin for atrial fibrillation and has been taking an antibiotic for urinary infection.

Full blood examination reveals:

In addition to parenteral vitamin K therapy, she should receive:

A. fresh frozen plasma.

B. cryoprecipitate.

C. recombinant factor VIIIc.

D. packed red cells.

E. platelet concentrate.

A

Answer: A

338
Q

QUESTION 51

A 48-year-old previously well man, currently smoking 10 to 20 cigarettes/day, presents with the progressive onset over several months of lethargy and headache. He takes no medications.

Examination reveals him to be plethoric and hypertensive with no other significant findings.

Which one of the following would most strongly support a diagnosis of polycythaemia vera?

A. Raised serum vitamin B12 level.

B. Elevated total red cell mass.

C. Normal marrow karyotype.

D. Elevated serum uric acid level.

E. Lowered serum erythropoietin level.

A

Answer: E

339
Q

A 19-year-old woman presents with recent-onset right leg swelling and pleuritic chest pain and is found to have iliofemoral thrombosis on Doppler ultrasound examination. Results of a ventilation-perfusion lung scan indicate a high probability of pulmonary emboli. She had been taking the oral contraceptive pill for the last three years but has now ceased. She is a non-smoker. There is no significant medical history and no known family history of venous thromboembolism.

The results of which one of the following investigations for an underlying hypercoagulable state are most likely to be affected by the presence of the extensive thrombosis?

A. Anti-phospholipid antibody screen.

B. Anti-thrombin level.

C. Assessment of prothrombin and factor V genotypes.

D. Activated partial thromboplastin time (APTT).

E. Full blood count including blood film.

A

Answer: B

340
Q

QUESTION 10

A 25-year-old man presents with recent onset of lethargy, fever, bruising and abdominal pain.

On examination, he is clinically anaemic and is noted to have scattered bruises, bilateral pleural effusions, an abdominal mass and cervical and axillary lymphadenopathy.

Full blood examination shows:

More than 90% of the nucleated cells in the bone marrow aspirate have the appearance shown below (examples indicated by arrows).

Coagulation parameters are normal.

The most likely marrow karyotypic abnormality is:

A. monosomy 7.

B. t (9;22).

C. 5q minus.

D. t (8;14).

E. t (15;17).

A

Answer: D. Burkitt’s lymphoma

341
Q

QUESTION 52

A 47-year-old man with severe haemophilia A and a high titre factor VIII inhibitor is scheduled for hip replacement surgery. The peri-operative replacement coagulation product of choice is:

A. recombinant activated factor VII.

B. recombinant factor VIII.

C. recombinant factor IX.

D. fresh frozen plasma.

E. cryoprecipitate.

A

A

342
Q

QUESTION 65

A 36-year-old man presents in a post-ictal state after an observed generalised seizure.

Full blood investigation shows:

The blood film is shown below.

The most likely cause of the peripheral blood macrocytosis is:

A. β-thalassaemia trait.

B. myelodysplasia.

C. alcoholic liver disease.

D. hereditary spherocytosis.

E. myelofibrosis.

A

Answer: C

343
Q

A 24-year-old woman, who has recently arrived in Australia from Vietnam, presents for evaluation of abnormal menstrual bleeding. There are no abnormalities on examination.

Results of investigations are listed below.

The most likely diagnosis is:

A. homozygous alpha+ thalassaemia (−α/−α).

B. early iron deficiency.

C. congenital sideroblastic anaemia.

D. sickle cell anaemia.

E. heterozygous beta thalassaemia.

A

A

344
Q

QUESTION 3

A 68-year-old man scheduled for herniorrhaphy presents for pre-anaesthetic assessment and is found to have non-tender cervical lymphadenopathy. Full blood examination shows:

The blood film is shown below.

With respect to the peripheral blood lymphoid cell immunophenotype, which of the following surface antigens is most likely to be co-expressed with CD19 (cluster of differentiation 19)?

A. CD3.

B. CD5.

C. CD8.

D. CD10.

E. CD34.

A

B. This is CLL as there are smudge cells on the film along with the other findings.

345
Q

QUESTION 16

In a 50-year-old woman, which of the following isolated laboratory abnormalities would be associated with the highest level of blood viscosity?

A. Serum immunoglobulin M (IgM) level of 40.0 g/L [0.5-3.0].

B. Peripheral blood leucocytosis of 25.0 x 109/L [3.9-12.7].

C. Haematocrit of 0.46 [0.32-0.42].

D. Serum immunoglobulin G (IgG) level of 40.0 g/L [7.5-15.6].

E. Peripheral blood thrombocytosis of 500 x 109/L [150-396].

A

A

346
Q

What is a Pelger-Huet anomaly?

In which condition is it seen?

A

Dumbbell-shaped bilobed nuclei on peripheral blood film

Myelodysplastic syndromes inc MDS, AML and CML

347
Q

QUESTION 44

A previously well 32-year-old man presents with progressive lethargy and no other symptoms. Physical examination shows him to have an enlarged spleen 6 cm below the left costal margin. There is no jaundice or lymphadenopathy and he is not clinically anaemic.

The full blood examination shows:

The blood film is shown below.

The most likely marrow cytogenetic abnormality is:

A. hyperdiploidy.

B. t(9;22).

C. t(15;17).

D. 5q minus.

E. trisomy 21.

A

B

348
Q

QUESTION 59

Megaloblastosis is least likely to be seen in which one of the following?

A. Transcobalamin I deficiency.

B. Pernicious anaemia.

C. Strict veganism.

D. Coeliac disease.

E. Post-gastrectomy.

A

A

349
Q

QUESTION 62

The delay in the onset of the anticoagulant effects of warfarin is most dependent on the:

A. loading dose of warfarin.

B. rate of catabolism of clotting factors.

C. rate of absorption of warfarin.

D. clearance rate of the S-isomer of warfarin.

E. clearance rate of the R-isomer of warfarin.

A

B

350
Q

QUESTION 22

A 75-year-old patient with chronic atrial fibrillation has been stable on warfarin 4 mg/day (international normalised ratio (INR) ranges from 2.1 to 2.5). The patient takes thyroxine 100 μg/day for hypothyroidism.

Which one of the following actions is most likely to potentiate the risk of bleeding?

A. Reducing the dose of thyroxine.

B. Increasing dietary vegetable intake.

C. Treatment with cholestyramine.

D. Treatment with omeprazole.

E. Treatment with amiodarone.

A

E

351
Q

QUESTION 33

A 72-year-old man with a history of chronic obstructive pulmonary disease (COPD) presents with an intercurrent lower respiratory tract infection. Apart from plethora and pulmonary signs associated with his respiratory disease, no other abnormalities are found.

Full blood examination shows:

The most likely cause of the elevated haematocrit is:

A. dehydration.

B. chronic hypoxaemia.

C. polycythaemia vera.

D. spurious polycythaemia.

E. low affinity haemoglobin.

A

B

352
Q

A 35-year-old man who is receiving erythropoietin (EPO) therapy for anaemia associated with end stage renal disease presents with lethargy. There is no obvious source of bleeding.

Blood investigations show:

The most likely cause of his anaemia is:

A. red cell aplasia.

B. reduced iron stores.

C. non-neutralising anti-EPO antibodies.

D. aluminium toxicity.

E. myelodysplasia.

A

A. Red cell aplasia. Due to NEUTRALISING EPO antbodies.

353
Q

QUESTION 46

Which of the following is most likely to prevent recurrent febrile non-haemolytic transfusion reactions in a patient requiring regular transfusion?

A. Warm blood to 37°C.

B. Leucodepletion.

C. Gamma-irradiation.

D. Cytomegalovirus (CMV)-negative donors.

E. Slow transfusion time.

A

B

354
Q

QUESTION 48

A 68-year-old man is sent for a second opinion regarding recurrent iron deficiency anaemia requiring blood transfusion.

Over the last three years, he has required an average of 4 units of packed cells every three months despite oral iron supplements. There is no history of melaena. Bone marrow examination is normal except for absent iron stores. Gastroscopy and colonoscopy have been negative on three separate occasions, the last being two months ago. Small bowel biopsy is normal. The patient had a barium meal and follow-through six months ago which was normal.

General health is good and he is on no regular medications.

Which of the following investigations is most likely to find the cause of his iron deficiency?

A. Small bowel enema.

B. Repeat gastroscopy and colonoscopy.

C. Enteroscopy.

D. Abdominal angiography.

E. Labelled red cell scan.

A

C

355
Q

QUESTION 66

A 28-year-old man presents with a fractured ankle requiring surgical fixation.

The most likely explanation for his anaemia is:

A. β-thalassaemia trait.

B. chronic blood loss.

C. anaemia of ‘chronic disease’.

D. congenital sideroblastosis.

E. sickle cell trait.

A

A

356
Q

QUESTION 84

A previously well 46-year-old woman presents acutely unwell with right iliac fossa pain and fever. A laparotomy is planned.

Which of the following is the most likely cause of the prolonged APTT?

A. Disseminated intravascular coagulation.

B. Heparin contamination.

C. Lupus inhibitor.

D. Factor XI deficiency.

E. Haemophilia carrier.

A

C

357
Q

QUESTION 93

A 16-year-old adolescent male presents with lethargy and lower respiratory tract infection. Physical examination shows him to be febrile, icteric and pale in addition to respiratory findings.

Full blood examination shows:

The blood film is shown below.

The most likely cause of the anaemia is:

A. paroxysmal cold haemoglobinuria.

B. glucose-6-phosphate dehydrogenase (G6PD) deficiency.

C. red cell aplasia.

D. cold agglutinin haemolysis.

E. sickle cell anaemia.

A

B

358
Q

QUESTION 9

Which of the following transfusion-related events is most frequently reported (episodes/unit transfused) with transfusion of unmodified red cells?

A. Acute haemolytic transfusion reaction.

B. Febrile non-haemolytic transfusion reaction.

C. Transmission of hepatitis B.

D. Graft-versus-host disease.

E. Transfusion-associated acute lung injury.

A

B

359
Q

QUESTION 18

A 67-year-old woman develops symptomatic anaemia and jaundice nine days after subtotal colectomy for adenocarcinoma of the colon, at the time of which she received a transfusion of 3 units of red cells. She is on therapy (which includes sulfasalazine) for ulcerative colitis.

Review of the pre-operative antibody screen shows this to be negative for red cell alloantibodies.

Her current full blood examination includes the following:

haemoglobin 82 g/L [120-155]

mean cell volume 99 fL [80-97]

reticulocyte count 162 x 109/L [8-104]

Her blood film is shown below.

The most likely cause of her anaemia is:

A. iron deficiency.

B. peri-operative blood loss.

C. red cell aplasia.

D. delayed haemolytic transfusion reaction.

E. megaloblastosis.

A

D

360
Q

QUESTION 41

During normal haemostasis, von Willebrand factor (vWF) plays a role in platelet adhesion/aggregation. During this process, which of the following is the primary binding site of vWF to platelets?

A. Collagen receptor.

B. Platelet glycoprotein Ib-IX-V.

C. Adenosine diphosphate receptor.

D. Platelet glycoprotein IIb-IIIa.

E. Ristocetin.

A

B

361
Q

QUESTION 60

Intravenous infusion of which of the following blood components is most likely to be associated with a septic transfusion reaction?

A. Red cell concentrate.

B. Fresh frozen plasma.

C. Pooled gammaglobulin.

D. Platelet concentrate.

E. Cryoprecipitate.

A

D

362
Q

What does antithrombin III do?

What is it’s clinical significance?

A

Deactivates factors 2 and 10, and thrombin.

Heparin potentiates it as it has a very high affinity for the part that inactivates thrombin.

LMWH has less of a charge and as a result, binds more to factor 10.

As a result, the thrombin clotting time is only affected by heparin.

363
Q

QUESTION 20

A 65-year-old man presents to the emergency department with ischaemic chest pain. No abnormalities are noted on examination apart from mild hypotension.

The most likely cause of the coagulation test abnormalities is:

A. disseminated intravascular coagulation.

B. administration of low-molecular weight heparin.

C. administration of thrombolytic agent.

D. lupus inhibitor.

E. administration of unfractionated heparin.

A

E

364
Q

QUESTION 36

A 30-year-old man has previously undergone splenectomy (six months prior) for chronic idiopathic thrombocytopenic purpura (ITP). Apart from occasional bruising, there are no haemorrhagic symptoms.

His full blood examination shows:

Repeated autoimmune and viral serology remain negative.

The most appropriate management of his thrombocytopenia is:

A. danazol.

B. immunoglobulin.

C. prednis(ol)one.

D. azathioprine.

E. observation.

A

E

365
Q

QUESTION 47

A 36-year-old healthy male with no siblings is found incidentally to have mild anaemia.

In addition to informing him of the diagnosis, which of the following is the most appropriate next step?

A. Globin gene analysis.

B. Provide information on partner screening.

C. Commence iron therapy.

D. Haemoglobinopathy screening for parents.

E. Gastrointestinal endoscopy.

A

B

366
Q

QUESTION 51

A 35-year-old man who is receiving erythropoietin (EPO) therapy for anaemia associated with end stage renal disease presents with lethargy. There is no obvious source of bleeding.

The most likely cause of his anaemia is:

A. red cell aplasia.

B. reduced iron stores.

C. non-neutralising anti-EPO antibodies.

D. aluminium toxicity.

E. myelodysplasia.

A

A